Этого треда уже нет.
Это копия, сохраненная 12 октября 2017 года.

Скачать тред: только с превью, с превью и прикрепленными файлами.
Второй вариант может долго скачиваться. Файлы будут только в живых или недавно утонувших тредах. Подробнее

Если вам полезен архив М.Двача, пожертвуйте на оплату сервера.
Тред тупых вопросов #64 #322871 В конец треда | Веб
Тред вопросов о жизни, Вселенной и всем таком.

Спрашиваем то, за что в других местах выдают путёвку в биореактор. Здесь анонимные ученые мирового уровня критически рассмотрят любые гениальные идеи и нарисованные в Paint схемы.

Предыдущий тут https://2ch.hk/spc/res/320340.html (М)
18 Кб, 1200x900
#2 #322880
Круговая станция на скорости влияет только на тех, кто стоит на ней? Если отскочить на 20 см от пола, будешь ли ты поддерживать ту гравитацию?
39 Кб, 700x348
Андрей эпп #3 #322881
В общем, суть вопроса моего тупая, наверное, но я пишу это чтобы быть +1 аноном, который постит хоть что-то и поддерживает наш спейсач алсо выхожу из ридонли узнать вот кое что. Марс является экзопланетой?
Вот есть же куча экзопланет, одну, недавнюю, прикрепляли в /b даже.

Неужели чисто теоретически нельзя представить там жизнь? Папоротники, черепахи, или кузнечики. Или они такие же как и Марс? Мол, на них можно ходить, танцевать хардбасс, но нельзя вырастить дерево и понюхать воздух.

Короче, мой вопрос это вообще вопрос? о возможной внеземной жизни. Она же должна быть там. Это логично. Если эти ЭКЗОПЛОНИТЫ из ряда Земных. Короче, мини-дискач, пожалуйста.

не поливайте жидкостью из гондонов, я тут пытаюсь постичь космос и делаю для этого всё
#4 #322882
Короче, мой вопрос это вообще вопрос? о возможной внеземной жизни
1,2 Мб, 3507x4102
#5 #322883
>>322882
Не читай спойлеры, они не для тебя.
#6 #322884
>>322880
Влияет не станция, а само наличие угловой скорости тела. Есть угловая скорость - есть центростремительное ускорение, есть центробежная сила. Станция нужна как опора, чтобы под действием этих сил не продолжать прямолинейное равномерное движение. Как бы такой барьер для инерции, а сама суть инерции и создает центробежную силу. Если подпрыгнуть, то никуда этот момент инерции не пропадет, по крайней мере не мгновенно. Поэтому после прыжка в достаточно крупных объектах такого типа вполне себе должно ощущаться полноценное подобие земного тяготения. Но если станция крохотная, радиусом метров 5, то прыжки могут стать порядком непредсказуемы из-за силы Кориолиса.
#7 #322886
>>322881

>постит хоть что-то и поддерживает наш спейсач


Ой блять, и без тебя нормально справляется. 5 лет назад тут постинг был меньше 24 в сутки и было лучше.

>Марс является экзопланетой?


Не является, гугли определение.

>Короче, мини-дискач


Проследуй куда-нибудь туда, не знаю куда. Здесь доска научными фактами и подкрепленными пластами научной логики гипотезами оперирует. В таких рамках это все, что тебя интересует, в духе экзотической фауны, очень уныло и плохо развито. Потому что соответствующей методологии не выработано, да и негде было ее вырабатывать. На твердые планеты в системе слетали - жизнь не нашли. Остается надеяться на спутники ГГ.
Вот тебе статьи для ознакомления, мог бы и сам:
https://ru.wikipedia.org/wiki/Парадокс_Ферми
https://ru.wikipedia.org/wiki/Внеземная_жизнь
https://ru.wikipedia.org/wiki/Жизнь_на_Титане
https://ru.wikipedia.org/wiki/Гипотеза_уникальной_Земли
https://ru.wikipedia.org/wiki/Возникновение_жизни
Как осилишь - задашь более конкретные вопросы. В общем и целом дискач на такую тему не для этой доски.
#8 #322893
>>322884
А почему звезды на евреев похожи?
#9 #322898
>>322871 (OP)
Дублирую из прошлой нити
Что будет с климатом Земли при наклоне оси в 0 градусов понятно - полное отсутствие сезонов.
Ад при уранианском наклоне с тропическим полярным днём смягчаемый морским климатом тоже хорошо описан. А какие условия будут при наклонах 30, 45 и 60 градусов? И как это повлияет на живое?
#10 #322912
>>322898

> 30


примерно как сейчас

> 45-50


большие сезонные колебания температуры, ледники только на вершинах гор и совсем крайних широтах, на полюсах, меньшие ареалы вечной мерзлоты, пиздец с погодой

> 60-65


ещё большие колебания температуры, тотальный пиздец с погодой, ледники исключительно на вершинах гор, везной мерзлоты нет совсем

> 85-90


ситуация описана в приливном захвате или длинном году Урана, в случае с землюшкой и Луной - всё намного хуже, т.к. сезоны будут
#11 #322914
>>322912

> 85-90


алсо забыл сказать что смена дня и ночи будет занимать ебанный нахуйблядь ГОД
#12 #322917
>>322914

> алсо забыл сказать что смена дня и ночи будет занимать ебанный нахуйблядь ГОД


хотя нет, немного серанул, но продолжительность дня в течении полугода будет довольно сильно колебатся
#13 #322930
>>322912
А может 30-35 КАК РАЗ для умеренных широт в плане смягчения климата? В карбоновом периоде вроде такой наклон был
#14 #322931
>>322930
нет, наоборот хуёво, летом жарче, зимой холоднее, погода более ебанутая
748 Кб, 512x512
#15 #322932
>>322884
А если такая станция, полностью искусственная, радиусом километров в 10-20, сколько по времени и насколько будет ощущаться это подобие гравитации?

А у меня такой вопрос. Представьте себе декартову систему координат, в 0;0;0 которой помещаем некое солнце. Через плоскость X;Y проходит орбита первой планеты, через X;Z - орбита второй, а через Y;Z - третьей. Все три планеты находятся в зоне комфорта, все три планеты имеют одинаковые размеры, имеют атмосферу, флору и фауну.
И на одной из этих планет оказывается ни с того ни с сего анон с котом.
Во-первых, как будет выглядеть движение двух других планет относительно наблюдателя на поверхности оной?
Во-вторых, как с помощью этих самых планет (а в это солнечной системе кроме солнца и этих трёх планет тел нет) определить положение себя, коли ты - этот самый анон - относительно широт: на полюсах ли ты, в умеренных ли, или на экваторе.
В-третьих, как определить, на экваториальной ты планете находишься, или ты накручиваешь круги вокруг солнечных полюсов.
В-четвёртых, каким будет вращение планет вокруг своей оси относительно плоскости экватора солнца (орбите какой бы планеты она не соответствовала)
Ну и в-пятых, более реалистичный вопрос: какова стабильность данной системы.
#16 #322933

> 3.


никак, нужно смотреть на солнце
#17 #322934
#18 #322935
>>322932

> 5.


никакой стабильности
#19 #322936
>>322933
И как смотря на солнце определить это? У нас в солнечной системе с этим всё понятно, а в указанном примере как это сделать?
#20 #322937
>>322936

> а в указанном примере как это сделать?


точно так же, определить направление вращения солнца
#21 #322939
>>322932

> 2.


относительно каких именно широт?
их у тебя много
#22 #322941
посоны, обьясните смысл(и историю возникновения) "поздний/ранний" относительно спектрального класса звёзд
#23 #322956
Оказывается на Луне хотят если и делать поселения, то лишь под слоем грунта. Это получается как кроты зарываться? Вам не кажется, что если посылать людей в космос, а потом они будут где-то там закапываться и сидеть под землей, то это будет возбуждать вопросы и теории заговора? Не, ну реально - отсылают людей на Марс или Луну, а они там в каком-то подземном бункере даже без окон сидят безвылазно и света дневного не кажут? Как вы убедите, что они вообще на другом небесном теле, а не где-то в бункере на нашей планете сидят и дурят народ? Я то верю, что они полетят, но вот если не доказать людям, то очень многие поверят в теории заговора опять.
#24 #322957
>>322956
Кто это хочет на луне заселять людей под реголит?
#26 #322960
>>322956

> Оказывается на Луне хотят если и делать поселения, то лишь под слоем грунта


хуя ты недогадливый, там жи радиация

> на Марс


там с этим немного проще

> Как вы убедите,


1. Зачем убеждать всяких шизиков?
2. имплуенг наземный город кого-то убедит
3. имплуенг на луне можно без использования йоба-телескопов( и даже с ними) увидеть хоть какое-то поселение
#27 #322962
>>322960
А зачем тогда вообще кого-то посылать? Как вам идея РЕАЛЬНО сфальсифицировать полет на Луну в мосфильме? Ну, в плане затраченных ресурсов и полученного результата? Это же реально сейчас с использованием CGI сделать, если мы захотим? Получится дешевле, не нужно никаких пусков ракет и новых исследований, не будет никаких рисков и жертв. А если еще и США убедить и Китай, то мы можем создать мощный медиапроект в каком-нибудь бункере в Гренландии - и даже крупные ученые будут думать, что мы реально на Луне все высадились и сидим там в какой-то пещере. Это доверие людей можно превратить в голоса.

Если не веришь, то ты шизик. Опа! Верь в нашу научную парадигму, никогда не сомневайся в наших словах, потому что "Мы (((ученые))) не обманываем друг друга!"
#28 #322963
>>322941
Гарвардская классификация, древняя, но в переработанном виде широко и всюду используется сейчас. Составлена в начале ХХ века, на Гарвардской обсерватории, на основе огромного числа спектральных снимков завезд.

О - В - А - F - G - K - M

>"поздний/ранний"


Когда-то считали, что звезды со временем просто остывают, поэтому О - В - А считались "ранними", то есть молодыми, а K и M - "поздними", то есть старыми.
Звезды класса O глаз видит голубоватыми, класса A — белыми, F — желтоватыми, G — желтыми, K — красноватыми, M— красными. То есть по снижению температуры. Сейчас поздний\ранний используется разве что только по привычке.
#29 #322964
>>322962

>Как вам идея РЕАЛЬНО сфальсифицировать полет на Луну в мосфильме?


Почитал чего интересного, про методы объективного контроля, просто для саморазвития.
#30 #322965
>>322963

> Когда-то считали, что звезды со временем просто остывают


спасибо
#31 #322966
>>322962

> США убедить и Китай


тогда выгоды ноль и огромные убытки
ну ты совсем зелёная зелень
#32 #322999
Что думаете?
https://lenta.ru/news/2017/01/24/rare_meteorits/
В статье говорится, что нынешние метеориты уже не те, и что раньше они разительно другими.
#33 #323000
>>322956
Ну телеметрию никто не отменял. Создать соответствующий приемник, который вычленит нужные параметры из сигнала, позволяющие уверенно заявлять, что сигнал идет таки с Луны, сейчас сможет и небольшая группа соответствующих специалистов, причем на голом энтузиазме. Но причем тут зогоебы? Все тайные заговоры давно стали специальной олимпиадой по наиболее изощренному "опровержению официальных данных". Ничто не помешает им думать на основе любых подтверждений дескать, записанные в земных павильонах кадры носит на Луну специальная АМС и передает оттуда. А на крайняк всегда можно придумать специальные стелс-шипы рептилоидов. Никто же не мешает некоторым сейчас думать, что МКС тоже на самом деле нихуя не летает.
#34 #323003
>>322999
выборка
алсо метеоритные потоки - штука не постоянная, где-то новое столкновение, где-то уже и нечему падать в землюшку то и т.д.
#35 #323005
>>322999
алсо это не новостной тред
#36 #323007
Объясните еблану на палках и желудях в чем принцип работы двигателя Альбукерке?
#37 #323008
>>322999

>В статье говорится, что нынешние метеориты уже не те, и что раньше они разительно другими.


Это в статье на Ленте говорится. В Статье в Nature говорится, что среди прочего обычного говна, которого находят валом до сих пор, попался ранее не похожий на остальные метеорит, который 470 миллионов лет назад был одним из доминирующих, их аж 1% падал, от общего говна, и что, возможно какая-то хуйня в космосе развалилась и высыпалась на Землю, потому что сейчас таких метеоритов не находят. Почему это важно? Потому что ВОЗМОЖНО это ведет к каким-то новым подробностям в истории Солнечной системы.
#38 #323010
>>323007
Ебаный стыд...
Во-первых, Алькубьерре.
Во-вторых, не упасть, а создавать вокруг корабля изнутри (иначе кина не будет).
В-третьих, НАСА искривляет пространство на десятимиллионную часть, контролируя это сверхточными интерферометрами, до самого варп-привода здесь - как до Антарктиды раком.
#39 #323012
>>323007
Принцип сжимать пространство перед кораблем, а за кораблем - разжимать. То есть корабль собственного движения не имеет, он катится по пространству, как на волне. Вот стоишь ты на ковровой дорожке и тебе надо попасть на ее край, вместо того, что бы идти к краю, ты начинаешь собирать ковер складками подтягивая к себе, а за спиной эти складки расправлять. Другой аналогии чет в голову не приходит.
#40 #323013
>>323010

>Во-первых, Алькубьерре.


Ах букакке? Алкодвери? Дольку веры?
не обращайте внимания, просто я дурачок и мне нравится коверкать Алькубьерре
#41 #323015
>>323010

> Во-первых, Алькубьерре.


ты совсем зелёный ещё, это местный мемас, это знать надо
#42 #323016
>>322871 (OP)
че там за залётный на третьем пике?
#43 #323017
>>323015
Как будто отвечать "Ебаный стыд..." и далее по тексту не местный мемас.
#44 #323018
>>323015
Ну ващет весь его ответ - такая же мемная паста, которая появилась чуть ли не сразу, как на этой доске Алькубьерре впервые был назван Альбукерке. И хотя надменный тон хуесоса, который это писал, абсолютно не осознавая мемной сущности форса, мне и самому не понравился, но все равно зашивайся, нюфак.
#45 #323019
>>323016
Голактека М87, там же написано.
#46 #323020
>>323017
как будто пояснять что это таки мемас не местный мемас
#47 #323021
>>323020
Это уже хуйня какая-то, а не мемас.
#48 #323022
>>323021
и это тоже мемас
и я тоже
#49 #323023
>>323020
Как будто что-то выраженное с помощью письменности или вербального общения не мемас.
#50 #323024
>>323023
только более-менее устоявшееся
#51 #323025
>>323018
Напомните, Альбукерке вроде из шапки одного из ликбез-тредов пошло?
#52 #323026
>>323025
Нет, было в треде. Какой-то анон написал. То ли по незнанию, то ли ради лулзов и понеслась.
#53 #323027
>>323026
Я выдвигаю теорию, что это было сделано ради лулзов, потому что мне кажется маловероятным, что можно настолько проебаться, что написать вместо Алькубьерре название города Альбукерке.
#54 #323028
>>323027
Ты недавно на двачах что ли? Тут такое пишут на серьезных щщах, что сидишь и охуевашь джва часа.
#55 #323029
>>323028

> Тут такое пишут на серьезных щщах, что сидишь и охуевашь джва часа.


чет рофляночки с этого
я ебанутый?
104 Кб, 600x400
#56 #323033
>>323026
Да, помню нюфаня исковеркал в вопросотреде. Полтора дебила подхватили и стали форсить "ололо мемос посоны".
Из-за этих дегенератов теперь уже все путаются.
>>323027
Вообщет, довольно сложная фамилия у этого мужика. немудрено напутать.
Miguel Alcubierre Moya
#57 #323034
>>323027
Это был залетный деградант, который насмотрелся Голливуда и невнимательно прочитал фамилию. А местные деграданты подхватили.
#58 #323036
>>323033
>>323034
А почему вам собственно с этого подпекает? Серьёзные вы какие-то.
#59 #323037
>>323025
Ну вот этот>>323010 ответ я впервые увидел точно как ответ на шапку, в которой было подобие фака и пузырь был выставлен научно обоснованным вариантом сверхсвета. Но оригинально да, по приколу кто-то ляпнул.
>>323034
Ну BrBa к голливуду отношение имеет довольно опосредованное. В том годувроде как раз в районе лета 13-го все это началось просто финальный сезон показывали, многих накрыло.
205 Кб, 400x300
#60 #323067
>>323013
>>323015
>>323037
Традиции спейсача.
#61 #323146
Есть чаво интересного почитать про зонд Галилео?
#62 #323148
Гравитация небесных тел продавливает просьранство, верно? Куда продавливает? Вселенная четырехмерна?
#63 #323152
>>323148

> Куда продавливает?


в пространство

> Вселенная четырехмерна?


одиннадцатимерна
#64 #323153
>>323152
Давай че по-проще, без струн
#65 #323154
>>323146
Нет, он утонул.
книг довольно дохуя написано, например
https://www.amazon.com/dp/1852333014
https://www.amazon.com/dp/0387987649
ни одну их них не читал, лежат пока


>>323148
Искривляет.
#66 #323169
>>322871 (OP)
Поясните за наблюдаемую вселенную. Чёт не соображу. Вот смотришь ты так далеко как только можешь и видишь прошлое. Вот я посмотрел на небо в 23 55 потом в 00 55. Что произошло за это время? что я увижу в 00 55? Это свет дошедший от более дальних регионов вселенной или чё ваще нахуй? Почему я не могу бесконечно улучшать телескоп и смотреть всё дальше?
#67 #323173
>>323169
до тела N 1кк световых лет.
Ты смотришь в 23 55 на него. и видишь то, что там происходило миллион лет назад в 23 55.
Смотришь на него в 00 55, видишь то, что там происходило миллион лет назад в 00 55.

Если посмотреть с тела M, до которого 60кк световых лет, на землю, то ты увидишь на земле динозавров сейчас(может не 60, не помню, когда они откинулись).

Ты можешь бесконечно улучшать телескоп и наблюдать. Но с земли наблюдать-моветон. Атмосфера загрязняет обзор. А вывод на орбиту километровых зеркал мы пока не могем
#68 #323174
>>323034

>мааам у нас тут сирьезная калабарация учоных мааам почему эти диграданты каверкают фамилииии

#69 #323189
>>323174

> сирьезная калабарация учоных


с миловым иминем, на зегундочку
#70 #323202
Чем обусловлено движение планет вокруг своей оси? Почему Венера и Уран вращаются с востока на запад?
#71 #323208
>>323173
А что происходит когда я смотрю на тело до которого 13.8 кк лет? На горизонт видимой вселенной. Откуда из за него идёт свет?
#72 #323209
>>323202
Хуй его знает, но скорее всего древнее OCHE BOLSHOE столкновение с чем-то ещё.
#73 #323217
>>323202

>Чем обусловлено движение планет вокруг своей оси?


Сохранением момента импульса. Первоначальные говна вращались, момент импульса при слипании говен никуда не пропадает, а передаётся телу. Каждая говнинка, прилипающая к планетеземали передаёт ей импульс, каждая планетезималь, при слипании в более крупное образование передаёт ему свой импульс, поэтому планеты вращаются. >Почему Венера и Уран вращаются с востока на запад?
Скорее всего в следствии крупного столкновения.
#74 #323219
>>323218

>изменения претерпевает материя, а не пространство


время изменяет ход, лоренц-фактор, вот это все - это материя или среда, быть может четверг, в говне моченный?
#75 #323221
>>323208

> Откуда из за него идёт свет?


человеческим, пожалуйста
#76 #323230
В современной науке в микромире хотя бы раз наблюдали баг времени, когда будущее наступило до настоящего?
8,1 Мб, webm, 854x480
#77 #323250
Как так то?
#78 #323253
>>323230
нет
#79 #323256
>>323250
Вы доебали уже, когда видео ускоряется, посмотри как меняется скорость полёта космонавта.
#80 #323257
>>323256
Как?
#81 #323258
>>323250
Это связано со спецификой зрения. Мозг короче отчетливей реагирует на определенные диапазоны скоростей. Например если ветер дует со скоростью 0.3 м/с и вдруг порыв со скоростью в 0,9 м/с, то человек не заметит, что ебать листва на деревьях задергалась в три раза быстрее или там облака полетели в три раза быстрее. Но разница скоростей допустим 100 и 150 км/ч двух автомобилей предельно очевидна. То есть как бы тебе объяснить, пакетик ускоряется, просто х2 пакетика неравнозначно воспринимается глазами, как х2 при махании рук. То есть блять, не знаю, ты ищешь какой-то скрытый посыл, или что? Типа в Голливуде проебались и не ускорили пакетик? Или что сам космонавт на зеленом фоне снимается? Ну так ты погляди, они ж даже ебаные тени в таком случае для тебя нарисовали, хуле выебываешься, жри что дают, ЗОГ лучше тебя знает.
#82 #323259
>>323257
Никак, изменения скорости с которой он парит не заметно, только изменение скорости движений руками и последующим за этим дёрганием тушки.
#83 #323265
>>323258
>>323259
В принципе я присмотрелся, и вправду кажется что пакет летит быстрее. Но это похоже по большей части на силу самовнушения, я заставил себя поверить в то, что я вижу, что он летит быстрее и вращается быстрее но я понимаю что это в принципе не так и мужик этот скорее всего совсем не в космосе.
#84 #323266
>>323265
У тебя самовнушение на тему того, что он медленно вращается. Всякий раз как ускоряется видео и пакетик тоже.
Ты бы еще привел вебм, где Тимоти Пик на фоне синей стенки, за неё бы тоже обоссали.
#85 #323267
>>323266
Да меня не обоссали, просто здесь же раздел про космос, то есть люди заранее думают, что все эти трансляции и полёты это всегда правда и не фейк.
А я как бы докапываюсь до истины. Вращается пакетик быстрее или не вращается это спорный вопрос.
#86 #323268
>>323250
предлагаю дать мне ссылку на исходник видео, чтобы я замерил скорость пакета в отрезки времени с ускорением и без, потому что по этой лоурезной вебмке ничего не понятно.
#87 #323269
>>323217
А почему вообще началось это вращение? Вроде же все вращается.
#88 #323271
>>323269
Из-за изначальной неоднородности Вселенной. Куски говен притягивались друг к другу, но немного промахивались, и получалось движение не по прямой.
#89 #323342
>>323173
а если некий космохуй с планеты М запилит мега-супер-оптико телескоп, в который Земля будет видна с разрешением-приближением скажем ну как на гуглокартах, что этот космохуй в телескоп увидит?динозавров или что?
#90 #323343
>>323342
Их ещё разглядеть надо. Но точно увидит Пангею, которая уже давно рассыпалась на всякие Азии и Америки.
#91 #323349
>>323343

>Пангею


Подразумевается, что теория Вегенера верна
#92 #323351
>>323349
А какие аргументы против неё?
#93 #323352
>>323351
А какие за? Какому-то сумрачному гению привиделось что контуры континентов подходят друг к другу и понеслось. Интересно, он обнаружил, что палец подходит к анусу или нет?
#94 #323353
>>323352
А как же происходящий по сей день дрейф материков в противоположные стороны, вот это вот всё?
#95 #323354
>>323353
Снова веруны протекают.
#96 #323356
>>323353
Из-за криво сделанных ЖыПиэСовых спутников точки ползают по карте на расстояние меньше погрешности прибора -> континенты движутся! Охуенная логика, чо.
#97 #323357
>>323354

>веруны протекают


Веруны в авторитет Вегенера?
#98 #323358
>>323357
С кем ты споришь.
#99 #323359
Почему в некоторых местах планеты нефти много а в некоторых почти нет?
#100 #323360
>>323354
Что, они толкают континенты тебе на зло? А как же ископаемые одного вымершего вида на разных континентах, одинаковая геологическая структура пород южной америки и восточного побережья Африки, данные паеомагнитных исследований?
>>323356
Странная погрешность, на два сантиметра в год направленно растёт.
#101 #323374
>>323267
Измерь скорость пакетика, измеряя расстояние между пикселями в каждом соседнем кадре. даже если его ускорение не заметно глазом. точные измерения его выявят (или не выявят, и тогда бугурта будет еще больше.)
#102 #323399
>>323360

>на два сантиметра в год направленно растёт


Ну и сколько десятков сантиметров погрешность у Глонасса/GPS? Вам хоть в глаза ссы "фактами", которых никак не могли обнаружить, а вы и рады.
#103 #323400

> заговорчики в /zog/


У вас целая доска на это есть. Даже быстрее чем эта.
#104 #323402
>>323356
Магнитные полосы по всему океану тоже погрешность? Наркоманы блядь.
#105 #323413
>>323356

> меньше погрешности прибора


если брать хуйню для васянов
а вообще пару миллиметров
#107 #323424
>>323422

> 2017


> не знать про gps-rtk

#108 #323430
>>322871 (OP)
Почему Путин на международных встречах не смеялся Обаме в лицо и не просил дать пруфы высадки на Луну?
#109 #323437
>>323430
Потому что американцы попросилих бы пруфы на Гагарина.
41 Кб, 500x367
#110 #323438
>>323402
Я думаю, для таких и Срединно-Атлантический хребет погрешность. Да и Земля плоская, хуле.
>>323437
А космоса, как все мы хорошо знаем, не существует.
#111 #323439
почему нету фото или видел полярной звезды с телескопа?
2,4 Мб, 2400x3000
#112 #323442
>>323439
Потому-что ты не умеешь в гугл. Вот тебе фоточка полярной и её подружек с Хаббла.
16 Кб, 300x230
#113 #323443
>>323439
С чего ты взял, что нет? И вообще что там рассматривать? Звезда как звезда, ничего особенного.
#114 #323444
>>323439
потому что звезды в телескоп видны, как звезды. Они неинтересны.
#115 #323446
>>323444
Ну во всякие Хабблы и прочие немного по-другому, из-за конского разрешения можно разделить тесную систему на компаньонов. Как с Сириусом, например. А так-да.
#116 #323448
50 Кб, 599x371
#117 #323454
https://naked-science.ru/article/sci/rossiyanin-pridumal-unikalnyy
1) Взлетит ли?
2) Играл ли автор проекта в КСП?
#118 #323455
>>323454
Какие-то ебанутые маняфантазии.
#119 #323459
>>323399
Есть несколько методов геодезического мониторинга ващет, не только спутниковый. Олсо ты точность навигации по коммерческим бюджетным приемникам берешь? Просто есть несколько частот. Вот платная, которую используют по лицензии геодезические приемники, дает отличную точность в пределах нескольких миллиметров. Все это правда на небольших расстояниях, сложно сказать, как там на материковых. Но все таки ты достаточно подкован в вопросе, точно знаешь, какие методы контроля используются и как они сравниваются, методику расчета погрешностей, и прочее? Или просто пиздишь "на глазок"?
#120 #323489
>>323459
Нашёл что спрашивать у транслирующего свой манямирок завсегдатая зогопараши. Нахуй ты вообще его кормишь?
#121 #323551
Голубцi, поясните, если нам надо пустым Союзом забрать ребят с мкс, она полетит строго на автоматике и gps или порой будут её подруливать на пару метров вправо из ЦУПа?
Можно из МКС управлять стыковкой внешнего объекта, ну то есть ребята за пару сотен километрiв берут управление на себя?
#122 #323553
>>323551
Союз, как и Прогресс могут в полет и стыковку на полном автомате.

>ну то есть ребята за пару сотен километрiв берут управление на себя?


Нахуя?
#123 #323554
>>323553
Им виднее, чем телеметрии с земли.
1,6 Мб, 346x195
#124 #323555
Итак. Освоивший космос человек прилетает на планету земного типа, с пригодной атмосферой, климатом, флорой и фауной, имеющим даже те структры ДНК с теми же аденином, гуанином, тимином и цитозином. Всё бы хорошо, но ДНК на той планете у всего и вся "отзеркалены" по отношению к ДНК чоловiка, то есть ДНК живности тут зеркально изомерны. Ясно, что на это планете человеку ни один вирус не страшен, вот только тут ни чего, что он бы переварил нет, ведь насколько я знаю, невозможно расщепить пищу на атомарный/маломолекулярный уровень еду, что перестроить её правильным для усвоения клеток образом ДНК - так ли это?
А если на этой плаете посадить семена, привезённые с собой, дадут ли они правильную флору?
#125 #323558
>>323555
>>323555
Естественно, если компост/почву с собой привезешь.
#126 #323560
>>323551
1) gps там не используется толком, нет особого смысла, т.к. координаты на орбите считаются иначе: баллистики на земле измеряют скорость и дистанцию радарами и дальномерами, а также инфу дает инерциальная навигационная система. Хотя вроде в новых модификациях (МС) глонасс все же запилили, как один из источников.
2) Как уже сказали, штатная стыковка - автоматическая, вмешиваться нет нужды. управлять с земли кораблем можно, однако не всегда есть радиовидимость с наземным пунктом. На МС есть антенна системы Луч (сеть релейных спутников), с ней не обязательно иметь пункты связи повсюду, но у неё тоже ограниченная видимость. Насчёт возможностей операторов ЦУПов (их несколько) - они очень широкие, например они иногда выполняют рутинную работу роботами-манипуляторами, пока космонавты спят или заняты.
3) управлять кораблем с МКС тоже можно.
#127 #323563
>>323558
А местная, разве не подойдёт?
#128 #323585
>>323563
Подойдёт, но есть нюансы. Особенно по части симбиотических отношений между некоторыми растениями и микроорганизмами.
56 Кб, 1024x654
#129 #323588
Как далеко от планеты может улететь пыль с этой планеты? Она может покинуть солнечную систему?

На поверхности МКС кажется нашли какие-то водоросли, а если так, то пыльца точно могла залететь. Значит они поднялись высоко. а потом их солнечный ветер типа хвостом может сдуть? Вопрос задаю потому что интересно, если серая слизь пожрет все на Земле, то сможет ли её снести в космос и куда-нибудь в другие звездные системы.
#130 #323591
Можно ли читать книгу в космосе просто при свете звезд?
#131 #323592
Если припарковать комету на орбиту Земли, то она будет своим хвостом создавать кольцо вокруг Земли и её будет видно долго даже днем? Как долго например комета Галлея может таять и испускать хвост в дневном небе вокруг Земли?
#132 #323593
>>323588
Пыль не очень далеко. Для съеба из СС нужно придать очень высокую скорость. На солнечный ветер надеяться тоже не стоит. С расстоянием он ослабевает, пока не упрется в гелиопаузу.
#133 #323594
>>323591
Конкретно где в космосе? Он сука большой и очень не однородный на человеческом масштабе
6,4 Мб, 4000x4740
#134 #323595
Вокруг нейтронных звезд получает невозмжна никакая жизнь или технология? Типа звезда создает сферу смерти из радиации, которую нужно облетать? Сколько вот таких сфер смерти в Галактике? Одни от нейтронных звезд, другие от сверхновых звезд, другие вблизи черных дыр. Еще какие сферы смерти есть?

Это как игра в сапер такая получается. Даже если жизнь или цивилизация где-то возникнет, то со временем по соседству может появиться какая-то хуйня, которая вовлекает живую планету в сферу смерти и убивает всё.
#135 #323596
>>323594
ну в нашем рукаве галактики наверное.
#136 #323597
>>323593

>в гелиопаузу


а какие-нибудь течения есть за пределами солнечной системы? например у нашего солнца есть своя гелиопауза как ты сказал? У других звезд тоже есть такие же, а между ними происходит взаимодействие, как между двумя пузырями, положительные и отрицательные заряды задают направление и вот уже есть какое-то течение, пыль приобретает еще одну скорость и вектор и летит куда-то по своим делам, пока не встретит комету или другое тело, которое занесет её ближе к другой звезде.
#137 #323598
В галактиках квазарах вообще возможна жизнь или технология, или же радиация из центра убивает всё?
#138 #323599
>>323596
Конкретнее.
#139 #323600
>>323591
В окрестностях Солнца, затенив его зонтиком - можно, хоть и темновато.
#140 #323604
>>323598

>радиация из центра убивает всё


this

>>323595
Активно излучающей неведомой ёбаной хуйни вообще навалом. Вон спутники Юпитера непригодны для жизни, т.к. находятся в его радиационных поясах. Проще перечислить где не лучирует, чем где лучирует. Космос вообще довольно враждебная среда, а тем более сгустки материи в нём - всё что связано с большими размерами/массой, обычно связано и с большими излучениями. Кто-то пытается насайфачить небиологическую жизнь, однако принцип все равно один - сильные излучения усложняют существование любых сложных структур.
#141 #323606
>>323585
Какие нюансы, например?
#142 #323607
>>323606
Например с грибами.
http://landdesigno.com/simbioz-rasteniy-i-gribov-article133.html
Микроорганизмы в корнях бобовых как еще пример. Растения при помощи бактерий регулируют химию почвы.
Для опыления цветов нужны насекомые.

Ну и еще вспомни, что даже у тебя в кишечнике существуют микрофлора, которая помогает в пищеварении. Все переплетено.
10 Кб, 1440x832
#143 #323610
У меня такой вот тупой вопрос: куда делась нулевая? В остальных разделах всё в порядке.
#144 #323611
>>323610
у меня такое же было, но сейчас вроде прошло.
#145 #323613
>>323611
Хм, как только запостил, у меня тоже заработало. Какая-то защитная система против ридонли чтоль, лол.
#146 #323614
По вечерам в окошечке наблюдается некий объект.
В безоблачную погоду, но с нашим городским светозасорением (или как там его) звезд не видно от слова совсем, а эта штука отчетливо видна.
Белая, не мерцает, размером со звезду, в игрушечный сорокократный телеском не выказывает признаков увеличения - далеко, то бишь.
Но на звезду не очень похоже - яркая больно. Светит точь-в-точь как самолет, только не мерцает и не двигается.

Вопрос - что это за поебень? Какая звезда? Или вообще что-то искусственное?
#147 #323616
>>323614
Скачай стеллариум и посмотри. Венера скорее всего, как подсказывают мне из отдела гадания и телепатии.
#148 #323622
>>323616
Спасибо, забыл о существовании таких хреновин.
Нашел в интернете оную карту, она сказала, что это действительно Венера.
Не знал, что звезда может быть настолько яркой.
5 Кб, 166x150
#149 #323627
>>323622
Но Венера не звезда.
#150 #323637
>>323627
вопрос терминологии веры
#151 #323656
>>323622
Кстати, рядом с ней сейчас Марс виден. Также пробивает даже мощную засветку, но не так впечатляюще
#152 #323659
>>323656
Прошлой зимой ночью одновременно ещё и Юпитер было видно. Я охуевал возле окна, когда провёл воображаемые линии их орбит относительно моей точки наблюдения и понимал, что вижу целый кусок солнечной системы невооружённым взглядом.
#153 #323715
>>323459

>ненастоящий GPS


Все ясно
398 Кб, 2760x2280
#154 #323717
не помню уже что конкретно за вопрос был и как давно его спрашивали, но вот ответ, пусть лежит здесь
5,3 Мб, webm, 640x360
холодный пот по спине пробежал #155 #323730
Что за хуйня? Откуда эти кадры? Оба шаттла пизданулись в атмосфере: один при старте, второй при спуске.
А тут в невесомости в безмолвии плывут обломки шаттла. Че за ебань? Откуда кадры?
86 Кб, 458x325
#157 #323734
>>323730
>>323732
Странно как-то всё. Поехавшие зогачеры усираются в попытках найти следы фальсификаций в реальных сьёмках с Луны и МКС, в то время как все остальные не в состоянии отличить средненький CGI с абсолютно плоской картинкой и рандомно влепленными "повреждениями", не учитывающими реальную конструкцию корабля, от реальности. Даже зная о обстоятельствах аварий, произошедших в атмосфере. И наверняка догадываясь, что рассыпавшийся при спуске корабль на орбиту вернуться не может от слова никак. Уже хотя бы потому, что спускается он только погасив орбитальную скорость. Я даже не знаю, как такое называется. Синдром Дауна?
И ладно там ещё на двачах, у нас совсем рядом ферма по выращиванию дебилов под названием b, но вы вот тут комменты почитайте:
https://www.youtube.com/watch?v=ST5ks0B2Gwc
Хозяин канала, судя по всему космической направленности между прочим, ещё и писал, что он долго сомневался в подлинности видео, но решил всё-таки выложить. Да ещё и написал, что это со дна достают обломки Челленджера. Дегенерат блядь. Просто пиздец.
#158 #323735
>>323734
О, я тоже видео перепутал, я тоже дегенерат.
#159 #323746
>>323734
А ты прикинь, что Колумбия не при входе в атмосферу пизданулась, а на орбите. И это реальные съемки. Как бы ты их скопрометировал? Рассказал бы что это сиджиай графен, наверное. Я бы так и сделал.
#160 #323749
>>323746

>А ты прикинь, что Колумбия не при входе в атмосферу пизданулась, а на орбите. И это реальные съемки.



Не стану толкать лекцию по физике, которую ты прогуливал в школе, ибо бисер закончился. Просто хочу спросить, почему у твоей Колумбии на крыле Эндевор написано?
3,8 Мб, webm, 1024x576
#161 #323751
>>323749
Ну давай включим логику зогачера и разберём по частям тобою написанное:
На орбите произошла авария космического корабля Колумбия, весь экипаж погиб. Власти решили скрыть это, потому что так надо. В космос был выведен челнок Эндевор, чтобы собрать останки экипажа и корабля, после чего спрятать их на Луне.
Но эта была многоходовочка, чтобы заманить на борт астронавта, который будучи ребёнком случайно увидел секретный масонский ритуал ZOG, когда пошёл поссать и ошибся дверью. Мировое правительство долгие годы содействовало его обучению и продвижению в астронавты, после чего устроило нарушение работы баллонов с жидким вакуумом на борту Колумбии, чтобы отправить его в этот роковой полёт.
Когда корабль вышел на рассчётную орбиту и приблизился к осколкам Колумбии, в двигательном отсеке начал крутить лом затаившийся там специально обученный смертник из Моссада, в результате чего Эндевор начал вращаться, командир экипажа уронил зубочистку на приборную панель, случайно нажав кнопки в заранее рассчитанной специалистами ZOG комбинации, в результате чего корабль уничтожила ионная пушка, установленная на спутнике системы орбитального подавления человечества.
Эти кадры были сняты вторым смертником Моссад, который ждал корабль в условленном месте орбиты, чтобы предоставить отчёт. После этого он поместил видеоплёнку внутрь себя, обмотался суперсекретной фольгой и опустился на землю, сам при этом сгорев. К сожалению, плёнку нашёл и оцифровал один из посетителей форчана, который случайно слил её в интернет вместе со своим паком трапов и порно с понями. Впрочем, за всеми кто её видел скоро вылетят чёрные вертолёты, вы все под колпаком блджад. Не стоило вскрывать эту тему.
#162 #323753
опишите эффект бабочки но в космосе
#163 #323754
>>323753

>опишите эффект бабочки но в космосе


Скажи по-человечески, что тебе нужно?
#164 #323755
>>323754
какую нибудь прикольную взаимосвязь между космическими событиями, любая фантазия
#165 #323756
>>323755
С фантазиями тебе в сайфай, дорогой.
#166 #323757
>>323756
сап анон
#167 #323758
>>323753

> эффект бабочки но в космосе


большой взрыв -> твой тупой реквест
расписывать не буду
#168 #323759
>>323758
мой тупой реквест -> большой взрыв
так прикольнее, но не физично
#169 #323772
>>323755
Ебнула где-то сверхновая, нанесла тяжелых металлов в эту систему. А теперь тут лысые обезьяны уранием друг другу сраки грозятся подорвать, да золотыми цепурами на шее меряются. Конечно сверхновая - не бабочка, и масштаб не тот, следовательно прямого соответствия нет, но бабочки в открытом космосе не живут.
1 Мб, 2963x3538
#170 #323777
>>323772
Ну это как посмотреть.
#171 #323779
Почему не запилят такую большую пушку, которая будет спутники и грузы всякие задёшево на орбиту забрасывать?
#172 #323780
>>323777
В Бабочке белый карлан сидит. Сверхновая там не аллах-бабахала.
#173 #323781
>>323779
Потому что спутники нужно делать из йобида чугуния. Иначе они не переживут такого ускорения, не говоря уже о технических сложностях изготовления и применения таких пушек. Короче, пытались и много, но не взлетело.
#174 #323782
>>323781
Можно что-нибудь попроще запускать. Там нямку для МКС. А космонавты её ловить будут.
#175 #323783
>>323782
Вторая трудность-это невозможность из пушки заебенить круговую орбиту. Снаряд должен уметь корректировать траекторию. Далее возвращаемся к пункту номер 1.
#176 #323784
>>323780
Ну это просто было к "бабочки в открытом космосе не живут". Так-то да.
#177 #323786
>>323783
Ну может там не так уж всё плохо для техники? Вон у мюрриканцев есть управляемые танковые снарды. Им норм, что их из пушки стрелячют.
#178 #323787
>>323786

>Вон у мюрриканцев есть управляемые танковые снарды.


Что еще у них есть? Розовые единороги?

>Им норм, что их из пушки стрелячют.


Принеси хоть один управляемый боеприпас, который выстреливается на 8 км\с
#179 #323788
>>323786

>Ну может там не так уж всё плохо для техники


Проще всего какую-нибудь хуйню как попугай повторять. "У омериконцев вот ето есть и то есть...я по пиздиливизорю слыхал..в угарной силе"
Какие конкретно есть у них "управляемые танковые", мм? Подкалиберный БОПС выдающий ~1,5 км/с у них "управляемый? Или какой нибудь гаубичный, еле выдающий 800 м/с, и при этом выдерживающий перегруз при выстреле от 4200 до 20 000 g?
Ну так ты кувалдой по своей "космической еде" и аппаратуре, что ты там запускать собрался, уеби - перегрузка на пару порядков меньше будет.
Ты вообще в курсе, что для того чтобы что-то на орбиту вышло, это что-то должно 7,9 км/с набрать, сказочник?
286 Кб, 871x700
#180 #323789
>>323788

>сказочник?


А я всё равно смогу.
#181 #323790
Да и что, тот чувак, который Саддаму обещал пушку, выводящую спутники, пиздел по вашему?
#182 #323791
>>323790
Он плохо кончил.
#183 #323792
>>323790
Пфф, плюю тебе на лысину с нашей лунной базы, построенной еще в 2015 году.
#184 #323794
>>322871 (OP)
Люди умирали в открытом космосе когда-либо?

Если нет то это довольно странно. В 60-е должны были подыхать все так же когда воздухоплавание осваивали.
#185 #323795
>>323794

>Люди умирали в открытом космосе когда-либо?


Нет, но в нескольких случаях были на грани.

>Если нет то это довольно странно.


Ничего странного, учитывая подготовку и перестраховки от отказов.

>В 60-е должны были подыхать все так же когда воздухоплавание осваивали.


Погибали, как правило еще на Земле, или по пути туда, или обратно.
339 Кб, 900x1276
ХАББЛ и ФОТКИ ПЛУТОНА #186 #323799
ананасы, почему Телескоп Хаббла может норм фоткать самые дальние уголки вселенной, но вот к плутону надо отправлять непонятную поебень, чтобы его сфоткать в норм качестве. Почему норм фотку Плутона не может сделать Хаббл?
#187 #323800
>>323799
Почему ты смотришь на гору/какой-нибудь здоровенный обьект вдалеке и не видишь на ней отдельных деревьев/деталей, мм?
Наверно потому, что излучение рептилоидов прожгло шапочку из фольги и задело глаза. Ну или потому-что дифракционный предел и угловое разрешение. Выбирай.
#188 #323801
>>323799
Потому что угловое разрешение.
#189 #323804
Почему до планет близких к Солнцу (Меркурий, Венера) добраться сложнее, чем к планетам более удаленным от солнца? Какие эффекты учитываются?
#190 #323807
>>323804
ежли совсем просто:
плонеты - это поебень, которая летает по кругу
землюшка - со скоростью ~30 км/с
внутреннее плонеты - шустрее, их надо догонять топливом или магическими орбитальными манёврами
внешние - медленнее, можна тормозить об юпитер
на самом деле всё сложнее и совсем не так, но остальные тонкости орбитальной механики ты можешь загуглить и сам, если не совсем тупой
#191 #323810
Почему когда варишь пельмешки, то они сначала лежат на дне кастоюли, а потом всплывают?
#192 #323812
>>323810
Нашёл что в разделе про космос спрашивать, наркоман ебаный.
Иди к кулинарам: https://2ch.hk/di/ (М)
#193 #323813
>>323810
1. При нагревании горячая вода устремляется вверх.
2. При закипании выделяются пузырьки, которые тоже идут вверх.
3. Холодный пельмень имеет меньшую плотность, чем разогретый.
4. Тесто довольно пористый материал, оно насыщается воздухом, и пельмень всплывает.
#194 #323814
>>323812
Да ладно, тут и без меня всякую хуйню постоянно спрашивают.
>>323813
Почему тогда это не работает с макарошками, например? Сами по себе они легче пельменей + целиком из теста состоят.
#195 #323817
>>323814

>Сами по себе они легче пельменей


Легче-тяжелее, ты умственно отсталый что ли? Имеет значение только плотность.
#196 #323823
>>323817
Про тесто ты сознательно проигнорировал?
#197 #323828
Если чёрная дыра даже свет не пускает, то я получается, могу к ней в плотную подобраться, харкнуть туда и мне ничего не будет? Раз электромагнитные волны там остаются, то и нагреваться рядом с ней я не должен, она ж почти ничего не излучает (я читал про то, что всё-таки какое-то излучение есть,но хз что это), так ведь?
#198 #323831
>>323828
А тип тебя только излучение в этом мире пугает?
#199 #323834
>>323831
Я спрашиваю конкретно про излучение, гравитацию оставим. Мне именно интересно, буду ли я подгорать рядом или нет.
#200 #323838
>>323834
Подгоришь от излучения пролетающего мимоговна, которое будет кричать OH SHI во всех спектрах.
#201 #323842
>>323838
Что за мимоговно?
#202 #323844
>>323842
Любая материя, которой не повезло там оказаться. Пыль, газ, астероиды, планеты, звёзды, боевые баржи Империума, Аллах.
#203 #323847
>>323844
не-не-не, я спрашиваю не в реальной модели а чисто обособленно от всего. Вот пустота. В пустоте чёрная дыра и я в дистанции плевка. Меня поджарит или нет? Мне не нужна реальность космоса, мне интересно именно в таком случае. Нет ничего, ни других звёзд, ни пыли, ни Аллаха с кораблями империума. Есть только я и ЧД.
#204 #323848
>>323847
Тогда нихуя тебе не будет, можешь плевать а твоя харча разгонится до 99.9999% скорости света и облучит тебя всем набором из спектра.
53 Кб, 679x451
#205 #323851
>>323848
Чёт проиграл с этой попытки самоубийства посредством плевка в чёрную дыру. Не плюй в гравитационный колодец, как говорится.
8,6 Мб, 5880x3136
#206 #323872
Почему у Дримчейзера нет переднего колеса? Он этой ногой будет царапать асфальт при посадке что ли?
#207 #323874
>>323872
кругом цветы, а я на лыжах!
Тебе асфальта жалко что-ли, или где?
#208 #323892

>


>>323807
А разве для того, чтобы подлететь ближе к солнцу, не надо снизить свою орбитальную скорость, а для того, чтобы от солнца отлететь, наоборот ее повысить?
#209 #323893
>>323892

> чтобы подлететь ближе к солнцу


в целом верно, но на плонетки не попадёшь
#210 #323905

>>undefined


Где то читал, что передняя часть корпуса планера очень сильно нагревается и резина не выдерживает таких температур, поэтому используют лыжи. Царапать асфальт будет только на последнем участке торможения, до этого будет кататься на задних колесах.
104 Кб, 1000x844
#211 #323908
>>323905
Почему все забыли про посадку на базе Эдвардс например, где асфальтом и не пахнет?
>>undefined
Ты что, двач сломал?
#212 #323912
>>323804
Кто тебе такое сказал? Нет такой зависимости.

До Венеры добираться на самом деле наименее затратно по характеристической скорости, меньше чем до Марса например. Да и лететь в 2 раза быстрее (гомановским переходом).

До Меркурия вот нужно относительно много дельты, да. Но не потому что он внутри траектории Земли, а потому что он энергетически далеко - надо погасить очень большую часть энергии орбитального движения, приданной кораблю Землёй, с которой он взлетел. Тормозить надо много, проще говоря.
#213 #323950
>>323872
Дохуя причин, вообще. Изначально планировалось колесо, даже рендеры вроде есть с ним, да.

Как объяснили лыжу:
1. Были проблемы с "определением характеристик шин после космического полета", решили что если одна из задних лопнет, это еще норм будет, а если передняя подведет - будет хуёво. Вообще странно, вот Х-37 летает, и ничего. Странно, что аэрокосмическая конторка не может как-то решить вопрос.
2. Можно сэкономить массу, и, что было важнее (как я понял из их писанины) объем в этом месте.

В общем, были проблемы, решили радикально, еще и массу сэкономили. НИКРАСИВА, это да, есть.
Почему задние тоже лыжами не заменили? Абсолютная потеря контроля на земле, да и садиться очень сложно было бы: задние стойки касаются поверхности первыми, на большой скорости.
Если присмотреться, лыжа действительно очень компактная и простая, вся покрыта ТЗП, собственно, даже испытания этих плиток на шкрябание полосы проводили, чтобы понять, как будут себя вести.
#214 #323954
>>323950

>Были проблемы с "определением характеристик шин после космического полета"


>вот Х-37 летает, и ничего


>Странно, что аэрокосмическая конторка не может как-то решить вопрос.


"Был получен опыт" ≠ "была получена основная составляющая ноу-хау конкретно этой конторой".
А то любят некоторые обобщать. (я не про конкретно тебя) Человечество научилось, ууу.
#215 #323966
Почему сегодня (31 января) все пбялились на Луну и какую-то яркую звезду рядом с ней? Ничего необычного вроде, часто такие яркие звёзды бывали. Но тут блять все на улице смотрят на неё и пальцем тыкают. Наверняка в сами что-то говорили об этом, иначе всем похуй было бы.
37 Кб, 576x419
#216 #323971
>>323966

>Венера


>яркую звезду



Спейсач проебан.
#217 #323973
>>323971
Второй раз за тред.
#218 #323974
>>323971

>Тред тупых вопросов


>Фейспалмить с залетных


>Сокрушаться про проебаный пейсач

#219 #323975
Что за взрывы первичных черных дыр? Как это вообще?
#220 #323977
>>323975
Если придерживаться ТБВ, то в начале существования Вселенной вещество было достаточно плотным, что бы в отдельных неоднородностях могли возникать чёрные дыры, не в следствии коллапса звёзд, а из-за уплотнения вещества. Таким образом, гипотетически, Вселенная могла получить совершенно невообразимое количество чёрных дыр совершенно произвольной массы. Начиная с массы нескольких атомов. Согласно гипотезе об излучении Хокинга дыры со временем испаряются, но крайне медленно, до наших дней могли дожить микроскопические дыры массой от нескольких тысяч тонн, или нескольких частиц. Согласно расчетам, при полном испарении чёрной дыры она должна неебически бабахать, выделяя довольно много энергии в виде электромагнитного излучения. Не смотря на то, что относительно прочих катастрофических явлений во вселенной такой взрыв все-таки не сильно мощный, но доступный для обнаружения, то астрофизики не оставляют надежд однажды такие взрывы первичных дыр найти.
#221 #323978
>>323977

> Согласно расчетам, при полном испарении чёрной дыры она должна неебически бабахать, выделяя довольно много энергии в виде электромагнитного излучения.


каким таким расчетам?
#222 #323979
>>323977

> Согласно расчетам, при полном испарении чёрной дыры она должна неебически бабахать


А откуда там столько энергии? Они ж по идее до нуля испаряются, нет?
#223 #323982
>>323978
>>323979
Hawking, S.W. (1977). The quantum mechanics of black holes. Scientific American, 236, p. 34-40.
#224 #323983
>>323982
Нашел, спасибо.
#225 #323984
>>323977

>но крайне медленно


Только если имеют очень большую массу

>до наших дней могли дожить микроскопические дыры массой от нескольких тысяч тонн, или нескольких частиц


>...интенсивность испарения нарастает лавинообразно, и заключительный этап эволюции носит характер взрыва, например, чёрная дыра массой 1000 тонн испарится за время порядка 84 секунды, выделив энергию, равную взрыву примерно десяти миллионов атомных бомб средней мощности.

#226 #323990
>>323984
Имел в виду, что они как раз доиспарялись до малой массы и должны взрываться.
#227 #323995
>>323789
Нет не сможешь.
>>323790
Он умер.
97 Кб, 800x533
#228 #323998
В Интерстелларе космонавты приземлились на планету, где 1 час равен 7 годам на земле, т.е. время замедлено в 61344 раза. Как это влияет на ночное небо? Будут ли звёзды в 61к раз ярче? Ведь, по сути, у звезд тогда есть гораздо больше времени осветить планету. Более того, такое охуевшее освещение может заменить солнце.
#229 #324002
>>323779
Сложность масштабирования легкогазовой пушки - ракету построить проще чем огромный многоразовый ускоритель, сириусли. Слишком высокие нагрузки на ПН при практически достижимой длине ствола (Жуль Верн предполагался в 3.5км вертикально в горе, т.е. сумасшедшая йоба - и то там предполагались нагрузки в 1000g). Огромные потери и сопутствующие перегрузки/нагрев в атмосфере. Невозможность перенаведения этой ебалы на другое наклонение, надо строить отдельную для каждого. А невозможность скругления как раз не была бы проблемой, если бы ПН это пережила.
#230 #324028
А если делать черную дыру не сжатием, а переносом энтропии из одного места в другое?
#231 #324032
>>323998
Солнце и звёзды двигались бы по небу в 61344 быстрее. Ну ты понел, какие нахуй звёзды - сплошное серо-голубое марево со светящейся полосой через весь небосвод(если орбита планеты стабильная и не сильно наклонена).
#232 #324033
>>324028

> делать черную дыру


/sf/

> переносом энтропии


хуя ты отбитый
#233 #324034
>>323998

> Будут ли звёзды в 61к раз ярче


будут, но частота тоже увеличится, хуй пойми как это будет наблюдаться
#234 #324060
>>323912
А планеты находящиеся вне орбитальной траектории Земли лучше поддаются? Или разницы особой нет? Просто этот анон >>323807 написал, что тормозятся легче.
#235 #324061
>>324060
Дело не в том, внутрь или наружу. Дело в разнице орбитальных скоростей и наклонений конкретных небесных тел.

Анон говорил о дополнительном гравиманёвре около Юпитера, чтобы сэкономить, я говорю лишь о стандартных гомановских переходах.

Чтобы сэкономить, можно много чего сделать - например об Венеру тоже можно делать гравиманёвры на пути к внешним планетам, и вполне себе делают многие АМС. И даже о Землю потом опять. И о любую другую планету, или хоть о все сразу, если их положение позволяет. (Вояджер, его даже назвали так) А на геостационарную орбиту Земли можно летать через Луну. А если не жаль сотни-другой лет, можно перелететь от окрестностей Земли в окрестности Юпитера буквально на одном пердеже в нужном направлении.

Орбитальная магия механика - запутанная штука.
#236 #324069
>>323753
Столкновение Земли с Тейей -> формирование Луны -> приливы -> достаточно широкие пологие береговые полосы между морями и сушей, которые и не суша и не море, полигон, облегчающий и ускоряющий для эволюционирующих организмов и растений задачу вылезти из воды на сушу.

Похожая система, но ближайший к твёрдым плонетам газовый гигант массивней и ближе все орбиты в долгосрочном периоде стабильны, при формировании системы отжирает на себя большую часть ледяных астероидов. В результате на одной из каменистых планет в центре зоне обитаемости нет общего океана, а только большие внутренние моря и озёра. Процесс абиогенеза в них занял более продолжительное время, причём пошёл разными путями в зависимости от широты и геологии - отсутствует Last Universal Common Ancestor, точнее у существующих видов их несколько, а не один. Среднесуточный перепад температур на поверхности значительно больше, многие виды приобрели способность временной практически полной остановки метаболизма при отрицательных температурах ныкаются в норах, на деревьях, высоко в горах.
#237 #324070
>>323814

>Почему тогда это не работает с макарошками, например?


Разная топология например, макарошка - сплошной тор, пельмень - многослойная сфера.
После оттаивания начинки унутри пельменной сферы образуется водяной пар и заполняет образовавшийся свободный объём между сжавшейся начинкой и тестом, дополнительно надувая тесто.
352 Кб, 673x379
#238 #324073
Предположим, звезду вместе с обитаемой планетой вышвырнуло какой-то неебической силой из плоскости Галактики перпендикулярно галактическому диску, и звезда оказалась от плоскости галактики где-то на расстоянии половины диаметра галактики. Смогут ли человеки, живущие на планете этой звезды, наблюдать на небе охуительную красоту наподобие пикрелейтед, или всё будет тускло, или вообще ничего видно не будет? И правильно ли я понимаю, что кроме этого на небе не будет ничего, кроме немногих крохотных туманностей, которые являются другими галактиками?
#239 #324074
>>324069

>бабочка


>ТейЯ


Ну нихуя ж себе.

>и растений


А растения у нас теперь не организмы? Или не эволюционирующие?
#240 #324075
>>324073

> Смогут ли человеки, живущие на планете этой звезды, наблюдать на небе охуительную красоту наподобие пикрелейтед, или всё будет тускло, или вообще ничего видно не будет?


Нет, не смогут, если у них не будет охуенного ночного зрения. Будет как сейчас Млечный Путь, только потусклее и на полнеба

> И правильно ли я понимаю, что кроме этого на небе не будет ничего, кроме немногих крохотных туманностей, которые являются другими галактиками?


Не совсем, насколько я помню, галактическое гало не совсем пусто, как миниум там будут шаровые скопления и чуть-чуть звезд.
#241 #324088
>>323779

>большую пушку


>спутники и грузы задёшево на орбиту


Тому що в продвинутых артсистемах и стрелковке часть заряда приделана к снаряду/пуле и движется вместе с ней в канале ствола, постепенно сгорая в движении и поддерживая условно постоянное давление во время движения в стволе.
Если перенести 100% заряда к полезной нагрузке, то можно будет совсем убрать ствол, избавиться от невъебенного ускорения на старте и использовать ЖРД с куда более высокими, по сравнению с метательными порохами, энергиями сгорания топлива.
#242 #324126
Анонасы - это фейк? Хули там хуйлуша круглосуточно снимает? https://www.youtube.com/watch?v=SF7FUU7CThs
#243 #324140
>>324126
Вы там ебнулись совсем? Что фейк? МКС? Камера? Ты плоскоземелец очередной пизданутый?
#244 #324145
>>324126
У тебя пластина в голове? Когда вы уже блядь начнете самообразовываться? Хватит быть дураками.
26 Кб, 600x405
#245 #324146
А вот такой вот вопрос. Часто на спейсаче пугают концом света от внезапно прилетевшем мощным гамма джетом на нашу родную планету, которая уничтожит атмосферу и испепелит человеков. Существуют ли методы обнаружения таких прямых джетов, чтобы точно можно было сказать когда наступит БП?
#246 #324152
>>324146

>чтобы точно можно было сказать когда наступит БП?


Перед взрывом сверхновой происходит так называемое нейтринное охлаждение. Так что обнаружение повышение потока нейтрино с какой-то стороны может быть свидетельством надвигающегося пиздеца. Однако, точное направление, при нынешнем развитии нейтринной астрономии, как и точная дата останется неизвестной. Учитывая, что джет имеет очень узкий раскрыв, быстро вычислить источник и понять, направлен ли полюс звезды четко в сторону Земли не представляется возможным.

>Существуют ли методы обнаружения таких прямых джетов


Нет, внезапный джет, он на то и внезапный, что пока не ебанет, ты его не увидишь. А когда увидишь будет уже поздно.
286 Кб, 871x700
sage #247 #324153
Пидорские джеты блять.
#248 #324157
>>324152
Читаю спейсач и не понимаю почему мы еще существуем.
#249 #324163
Карочи, анонасы, что выгоднее - дать ЧД испариться и ебануть на пол-планеты, или запустить ее в ядро, чтоб она там все распидорасила и поглотила?
#250 #324180
>>324163
Выгоду в чем ищешь?
389 Кб, 999x1260
#251 #324181
Если корабль на околосветовой скорости вьебенится в очень тяжелую частицу, то ему пиздец ведь? Получается создав вокруг Солнечной системы поля из тяжелых пылевых частиц, мы можем предотвратить вторжение незванных гостей? Что-то вроде известного ведра с болтами на орбите, только в большем масштабе?

Допустим у пришельцев даже есть небольшой лазер, но можно создать многослойные термостойкие частицы и труднообнаружимые радаром частицы? Тогда межзвездный корабль пришельцев сначала не обнаружит частицы во время на пути, а потом ему не хватит времени обработать их лазером?
#252 #324182
>>324181
Энергию капли малафьи, врезавшейся в корабль на околосветовой скорости можешь посчитать сам. Ну и ничто не мешает алиенам вторгаться в твой манямир солнечную систему на досветовой скорости.
#253 #324184
>>324182
ну если мы примерно можем предсказать маршруты, то не потребуется создавать всю сферу, а лишь закрывать направления.

Энергия просчитывается от массы частицы плюс скорость корабля. Так что если частица тяжелая, то энергия столкновения должна быть как взрыв мощной бомбы. Какие-нибудь частицы с ядром из обедненного урана не канают? Если там много частиц таких, то его как при наезде на барьер-блокиратор должно расхерачить.
108 Кб, 504x566
#254 #324185
С другой стороны, такие поля частиц выгодны для удержания не пришельцев, а населения внутри Солнечной Системы. Пососи Маск и прочие мечтатели убежать от Земли к другим звездам. Никого не выпустим, будете гнить в солнечке с нами, никаких вам Оснований или отколовшихся цивилизаций бродяг.
#255 #324186
>>324184

Если кто-то умеет летать на околосвете, то и с такой хуйней должен справляться
#256 #324187
>>324184
>>324185
По-моему тебе даже по меркам этого треда можно выдавать путевку в биореактор.
#257 #324188
>>324187
аргументируй
#258 #324189
>>324188
Аргументировал тебе за щеку. Твоим алиенам с околосветовой скорости надо будет тормозить задолго до солнечной системы, если они не хотят превратиться в паштет внутри своей хуитки. А на маленьких скоростях на твои частицы будет всем похуй.
#259 #324190
>>324186
Сам факт владения одной технологией не означает автоматически, что в 100% случаев у них есть решение для всех проблем. Это логика типа "ну раз они в околосвет могут, то они и живых желейных человечков могут делать". Да блять, при желании могут, но между желанием и реализацией находится какой-то период решения проблемы и создания технологии.

Представь что мы уже посылали около 10 кораблей к ближайшим звездам, а когда послали к 11 звезде, то внезапно натолкнулись на такое препятствие. (как бы они соответствуют условию задачи - умеют в околосвет уже, да) Экипаж корабля в криосне, кораблем управляет ИИ, которые не успевает вовремя среагировать, его лазер не может расплавить слой наномин, кораблю пизда. А в звездной системе 11 пришельцы получают сигнал, что их план сработал.
#260 #324192
>>324189

>надо будет тормозить задолго до солнечной системы


Ну и разместим минное поле в этих зонах. Нанотехнологии, сынок. Можем создать фабрики, которые будут их мегатоннами выплевывать в потенциальные направления.
#261 #324194
>>324192
Разместил свой зонд в твоей защечной зоне.
#262 #324196
>>324181

>Если корабль на околосветовой скорости вьебенится в очень тяжелую частицу, то ему пиздец ведь?


В одну - нет, с хуя ли?

>Получается создав вокруг Солнечной системы поля из тяжелых пылевых частиц


Каким образом?

>можем предотвратить вторжение незванных гостей?


Да кому мы нахуй нужны.
#263 #324198
>>324196
В одну нет, но если он столкнется с большим облаком, то да.
Если не вокруг всей Солнечной системы, то хотя бы прикрыть направления. Можно периодически создавать такие облака при помощи нано-фабрик в поясей Койпера. Их точное местоположение будет известно лишь правительству, так что можно создать определенные риски для незваных гостей и несанкционированных беглецов. Межзвезный трафик в любом случае надо будет как-то регулировать, это можешь даже не сомневаться.
#264 #324202
>>324198

>то хотя бы прикрыть направления


Известны точные направления, с которых прибудут гости, или есть какая-то причина, которая не позволит облететь эти заградительные поля?

>Можно периодически создавать такие облака


Ты имеешь весьма слабое представление о космическом пространстве и его размерах.

>при помощи нано-фабрик в поясей Койпера


У нас есть серьезная проблема просто отправить аппарат до пояса Койпера, а уж размещать и снабжать фабрики вообще нет никаких возможностей.

>Межзвезный трафик в любом случае надо будет как-то регулировать, это можешь даже не сомневаться.


Единственное, в чем я не сомневаюсь, так это в том, что ты перепутал спейсач с сайфаем.
#265 #324204
>>324202

>Известны точные направления, с которых прибудут гости


гипотетически. Можно выделить направления с которых они НЕ будут подлетать - оттуда где у нас поставлены поля. У гипотетического командующего будет больше информации. Облететь поля на околосвете представляется затруднительным для определенных кораблей в силу перегрузок на корпус и содержимое.

>У нас есть серьезная проблема просто отправить аппарат до пояса Койпера


Я не говорил, что делать это уже завтра. Это идея для цивилизации уровня где-то 1.4 по Кардашеву. Дело лишь в принципиальной возможности. Единственная проблема которую я пока вижу это недостаток материала.

>что ты перепутал спейсач с сайфаем.


Ну тред для тупых вопросов, так что норм, мне интересно что скажут аноны.
4,1 Мб, webm, 320x240
#266 #324207
>>324181

>Если корабль на околосветовой скорости вьебенится в очень тяжелую частицу, то ему пиздец ведь?


Нет. Даже у частиц сверхвысоких энергий, время от времени прилетающих из космоса, энергия не больше нескольких джоулей. Это как из воздушки залепить. правда с другими последствиями вроде радиации, и относительно высокой проникающей способностью

>поля из тяжелых пылевых частиц


Ты разберись, поле у тебя или частицы, какая-то каша у тебя в голове. И что ты подразумеваешь под частицами - сверхтяжелые элементарные частицы, или пыль. Сверхтяжелые элементарные частицы потому тяжелые, что движутся очень быстро, со скоростями, очень близкими к световым. 0.9999... че-то там, чем больше девяток тем тяжелее. То есть в них запасена огромная (для элементарных частиц, не для нас!) кинетическая энергия. И уж тем более поле что ты под этим понимаешь? Поле как форму материи? группу макрообъектов? создать из них нельзя. С пылинками то же самое, чтобы они что-то повредили, столкновение должно произойти на достаточно большой скорости, плюс эффект зависит не только от энергии столкновения, но и от физических размеров.

>Что-то вроде известного ведра с болтами на орбите


Известное ведро с болтами на орбите - хуйня собачья для доверчивых космооперных геев. Хинт: орбита бывает не одна, а баллистика - очень сложная штука. Заполнять всё пространство даже вокруг Земляшки ты натурально охуеешь. (и лишишься космонавтики в результате)

>межзвездный корабль пришельцев


>многослойные термостойкие частицы


>труднообнаружимые радаром частицы


>обработать их лазером


ayyy lmao :DDD
/sf/ где-то в той стороне ->
#267 #324210
>>324207

>С пылинками то же самое, чтобы они что-то повредили, столкновение должно произойти на достаточно большой скорости, плюс эффект зависит не только от энергии столкновения, но и от физических размеров.



Так в этом и идея. Они сгрупированны в облако/поле, тяжелы и имеют большие размеры, форму в виде треугольников (как стелс) чтобы дурить радар противника и рассеивать лазерное излучение. Скорость у них определенная есть, но скорость есть и у корабля пришельцев.
#268 #324211
>>324210
Уносите этого в сайфач.
#269 #324213
>>324204
Примерно 1,69635×10^12 км площадь сферы, где начинается пояс Койпера. Теперь возьми калькулятор и попытайся посчитать сколько нужно говна, что бы закрыть хотя бы четверть этой площади.

>Так в этом и идея. Они сгрупированны в облако/поле, тяжелы и имеют большие размеры, форму в виде треугольников (как стелс) чтобы дурить радар противника


Если ты думаешь, что облако говна можно каким-то образом скрыть от средств наблюдения - ты заблуждаешься, оно в видимом диапазоне будет блокировать часть света, что уже демаскирует его.

>рассеивать лазерное излучение.


>пиу-пиу


Уходи в сайфач, пожалуйста.
#270 #324219
>>324213

>1,69635×10^12 км площадь сферы, где начинается пояс Койпера


Это неверные данные, это не площадь, и даже не радиус. Чек ёр сорсес.
#271 #324227
>>324219
Да, что-то обосрался где-то.
Пересчитал получилось примерно 6.36*10^19
#272 #324228
>>324213

>>пиу-пиу


>Уходи в сайфач, пожалуйста.


А что сразу в сайфач? Межзвездный корабль должен быть оснащен лазером чтобы распылять естественную пыль на пути следования. Если же пыль будет не естественная, специально приспособленная выдерживать лазерное излучение, то у корабля могут возникнуть проблемы.
Это ведь не только мы можем эту хуйню создавать, мы можем на эту хуйню наткнуться, если её оставили предыдущие цивилизации. В космосе какой-то только хуйни может не быть, может там фабрики по производству пыли все еще работают, а цивилизации уже мертвы.
#273 #324232
>>324228

>А что сразу в сайфач?


>Межзвездный корабль должен быть оснащен лазером


А дятел оборудован клювом. Пиздуй нахуй со своими фантазиями, чепушило, если ты нормальным языком не понимаешь.
sage #274 #324239
>>324228

> А что сразу в сайфач? Межзвездный корабль должен быть оснащен лазером чтобы распылять естественную пыль на пути следования. Если же пыль будет не естественная, специально приспособленная выдерживать лазерное излучение, то у корабля могут возникнуть проблемы.


> Это ведь не только мы можем эту хуйню создавать, мы можем на эту хуйню наткнуться, если её оставили предыдущие цивилизации. В космосе какой-то только хуйни может не быть, может там фабрики по производству пыли все еще работают, а цивилизации уже мертвы.


Ты прав, тебе не в сайфач, в дурдом надо.
#275 #324240
Поясните про конденсат бозе-эйнштейна простым языком.
74 Кб, 377x341
#276 #324262
>>324240
Вещество при (почти) абсолютном нуле, отчего многие его атомы находятся в полном покое, просветляются и становятся буддами и вся эта ебала обретает особые свойства, обусловленные квантовыми эффектами.
#277 #324263
>>324262
Что за йоба свойства такие?
#278 #324281
Можно зайти в две двери одновременно, лул.
#279 #324290
Уважаемы. я у вас тут впервые. Причина по которой я явился на вашу многоуважаемую доску такова: В голову мне пришла тупая идея, заключающаяся вот в чем:

Что если построить огромную шахту для космической ракеты, но шахту не рыть в земле, а строить из бетона или ещё чего в высоту? и покрывать так километр за километром, грубо говоря, найти какую-то достаточно широкую возвышенность, положить на неё слой бетона(или ещё чего) площадью в Х и высотой в 25м, на него положить слой в Х-1% толщиной в 25 метров. в итоге к концу мы получим шахту высотой в 2.5км(можно укладывать большее количество метров, или сокращать не по проценту, а по пол процента, например)

Если запускать ракету из такой шахты, её разгонный потенциал должен ведь возрасти в разы, а траты на топливо значительно снизится или как?
Скажется-ли постройка такой шахты на цене запуска ракеты(если не учитывать цену постройки шахты)?
#280 #324291
>>324290
Что-то я не уловил суть твоей идеи, уважаемый. Каким образом шахта будет помогать взлёту ракеты?
#281 #324292
>>324291
Ну, я может дурак, не бейте.

Насколько я знаю, ракеты идут на реактивной тяге, что подразумевает что всякое говно, вылетая из сопел в низ, толкает отдачей ракету в верх. Соответственно, если рассеивание газов будет ограничено шахтой, то они смогут передать больше энергии ракете, или нет?

Ну типа как стволы у военной техники, чем длиннее ствол, тем выше ускорение снаряда и его пробивная сила(не всегда, но как правило)
35 Кб, 500x367
#282 #324298
>>324290
>>324292
Ничего не произойдёт. Разгонный участок очедлинный, а шахта короткая, и занимает одну пиздохулионную пути разгона. Полученный профит в разнице давлений ничтожен - даже если ракету смазать вазелином и загерметизировать шахту, там и без того разница давлений в дохуя атмосфер, у некоторых движков под пару сотен.

> километр за километром в горе


Охренеешь это строить, проще построить ракету.
#283 #324300
>>324298

>Разгонный участок очедлинный, а шахта короткая


Так я и хотел длинную шахту.

>пиздохулионную пути


докуда вообще ракету надо докинуть? до стратосферы? или куда?

>без того разница давлений в дохуя атмосфер, у некоторых движков под пару сотен


так в схеме с шахтой давление будет юзатся от уже использованных газов, а на открытой площадке движкам нужно будет его воссоздавать со скоростью, с которой газы рассеиваются в атмосфере. т.е. будет требоваться больше топлива?

Я-то думал, что это значительно сократит цену топлива, ведь большая часть топлива расходуется именно на разгон(насколько я слышал)
#284 #324302
>>324300

>докуда вообще ракету надо докинуть? до стратосферы? или куда?


До 7.8 км/с вбок на высоте 250км или выше.

>в схеме с шахтой давление будет юзатся от уже использованных газов, а на открытой площадке движкам нужно будет его воссоздавать со скоростью, с которой газы рассеиваются в атмосфере


Энергия сгорания топливной пары постоянна, неоткуда взять лишнюю. И она уже и так используется в ЖРД почти лучшим образом. Можно конечно оттолкнуться от земли (в виде шахты), но энергию-то ты все равно возишь с собой. Сопло Лаваля с конфигурацией под внешнее давление + камера сгорания с хорошим давлением уже и без твоей шахты обеспечивает почти оптимальное расходование энергии химической реакции сгорания данной пары. Можно еще чуть оптимальней, но профит будет маленький, а затраты большие.

Вот если бы ты предложил электромагнитный ускоритель, подпитываемый энергией извне... то мы бы опять пришли к из пушки на луну
#285 #324303
>>324300

>Так я и хотел длинную шахту.


Вырастут аэродинамические нагрузки на ракету и атмосферные потери, помимо всего прочего. Профита не будет.
#286 #324304
>>324302
>>324303
Я понял, спасибо за пояснение.

>Вот если бы ты предложил электромагнитный ускоритель, подпитываемый энергией извне


Насколько я понимаю, объект разгоняется до ёба скоростей, и дальше идет на инертной тяге до заданной точки? По моему, профтит тут только при доставке грузов, и то не всех, ибо людей от перегрузок нахрен убьет.

Всем спокойной ночи, и ещё раз спасибо за пояснения. .Ваш мимо из wm/.
#287 #324336
>>324300

>так в схеме с шахтой давление будет юзатся от уже использованных газов


Ну ты бы это, еще предложил нарезы сделать внутри шахты, шоб ракета как винтовочная пуля хуярила. Так-то знаешь ли даже подкалиберные боеприпасы свой "калиберный" диаметр только в полете теряют. Если диаметр ракеты не соответствует диаметру шахты, то профиты от всей конструкции вообще мизерны, только погружать само тело ракеты в облако раскаленных газов на всем пути. Это если только поршень специальный делать под днищем, и чтоб он в полете отваливался, но само требование наличия такого поршня убивает весь смысл затеи.
За технические стороны тебе уже пояснили, я лишь за здравый смысл пытаюсь пояснить.
#288 #324339
>>324290
Нет, чтобы выйти на орбиту, нужно набирать скорость вбок, а не вверх. Сам по себе набор высоты расходует несравнимо меньше топлива, чем набор горизонтальной скорости.
#289 #324341
>>324336

>Ну ты бы это, еще предложил нарезы сделать внутри шахты, шоб ракета как винтовочная пуля хуярила.


Тогда будет лететь строго вверх, а не по параболе.
#290 #324348
>>322871 (OP)
Планетные орбиты выравниваются в минимальном магнитном поле? То есть джеты солнца нас не заденут.
#291 #324349
>>324348
Какие еще джеты Солнца? Обколются своей марихуаной...

Джеты исходят из полюсов, планеты образуются в экваториальной плоскости.
#292 #324350
>>324349
А значит нас не заденут, я про то и имел ввиду.
#293 #324351
>>324350
Нас нечему задевать.
#294 #324353
Чисто теоретически, без учёта сложности проектирования и конструирования, возможно ли создать такое зеркало, которое бы держалось в заданной точке околопланетного пространства за счёт уравновешивания притяжения и давления солнечного излучения, с минимально возможной активной коррекцией?
#295 #324355
>>324353

>за счёт уравновешивания притяжения и давления солнечного излучения


Кек.
Есть точки, где уравновешиваются силы притяжения двух тел. Называются точками Лагранжа, они псевдостабильны, там размещают обсерватории с минимальной активной коррекцией.
28 Кб, 800x600
#296 #324361
>>324355
Если просто повесить зеркало в L4 или L5, то его снесёт световым давлением. И вообще я имел в виду что-то вроде пикрелейтеда.
#297 #324362
>>324361
можно и уже кажется такое предлагали для создания мифических роев Дайсона.
#298 #324363
>>324361

>что-то вроде пикрелейтеда.


Но зачем? Что бы убить у всего живого биоритмы, или перегреть планету к хуям?
#299 #324366
>>324362
Если мы дошли до роёв дайсона, то городить такую хуйню к планете уже в принципе не нужно.
>>324363
Чтобы разогреть выпавшую снегом на теневой стороне атмосферу у приливно захваченной планеты, например.
#300 #324368
>>324361
разверни большее зеркало и ебани его параболой
#301 #324370
Что там с VASIMR? Уже долетели до Марса за 40 дней?
#302 #324371
>>324370
Испытания на МКС протухли и их отменили. Теперь хотят испытывать в вакуумной камере в следующем годоу.
#303 #324372
>>324371
А что там вообще слышно насчёт его будущего? Есть какие-нибудь перспективы вообще, или откажутся возить с собой АЭС? Я так понимаю, что это его самая большая проблема, не?
#304 #324373
>>324372
Перспективы как обычно есть ровно настолько, насколько есть:

1) большие межпланетные нагрузки
2) способ генерации дохуя электричества и сброса дохуя тепла

То есть без пилотируемой миссии это всё имеет мало смысла, т.к. АМС не нуждаются в большой массе и быстрых маневрах. Вот если бы насо развивало свои межпланетные SEP/NEP модули, тогда да, а так всё пылится на полке, включая SAFE-400 и проекты по его апгрейду.
#305 #324374
>>324368
На картинке масштаб не соблюдён.
#306 #324377
Правда ли что умершие попадают не в мифический рай, а на специальную планету, которая по сути и является раем?
#307 #324379
>>324377
Ты "контакта" обсмотрелся там?
#308 #324380
>>324379
Просто часто в последнее время слышу когда одна бабка говорит что-то вроде "мой муж/сын/дочь в раю сейчас", а другая ей отвечает "учёные доказано, что рая не существует; они попадают на другую планету".
29 Кб, 402x410
#309 #324381
>>324377
Не совсем. После смерти мозга твоя личность умирает вместе с ним и ты исчезаешь навсегда. Правда люди в основной массе слишком трусливые, чтобы принять тот факт, что "я весь такой важный и значительный просто исчезну вникуда, а не буду в виде бессмертной души чё-то где-то делать", поэтому выдумывают загробную жизнь разных сортов и консистенций. И да, /sn и /re в другую сторону.
#310 #324382
>>324381
Есть мнение, что информация не может просто бесследно исчезать.
#311 #324383
>>324382

>информация


нет такой физической величины.
информация это сорт оф порядок.
а порядок вполне превращается в хаос.
#313 #324385
>>324384
Это очень сложный вопрос, не уверен что тебе на него здесь ответят.
#314 #324387
>>324382
Болезнь Альцгеймера не существует, или что?
#315 #324394
Не убедительно.
#316 #324398
>>324394
Теперь это убеждения-тред?
160 Кб, 600x332
#317 #324404
>>324377
Это не какая-то планета является раем, это Земля ёбаный ад. Льюис не даст спесдеть.
60 Кб, 992x206
#318 #324406
>>324404
И в чём заключается адскость нашей планеты?
#319 #324408
>>324406
Ну как, юдоль страданий, всё происходит через анальную жопу заднего прохода. Льюис писал "Космическую трилогию" без малого век назад, когда толком ещё не было известно об условиях на всяких марсах-венерах. Поэтому он со спокойной совестью сделал их обитаемыми. Такие дела.
117 Кб, 985x789
#320 #324409
>>324408
А почему мне не страдается-то? Вроде и жизнь совсем не сахар, но чот всё равно норм. Короче, хреновый ад какой-то, у Данте намного лучше было это всё.
А вообще, Льюиса я не читал, но если у него очередная петушиная экзистенциальщина, хэлл ис озер пипл или нытьё про упадок нравов - то пошёл он нахуй.
#321 #324411
>>324406

>И в чём заключается адскость нашей планеты?


В том, что вся жизнь основана на убийстве. Все друг друга пожирают, чтобы выжить.
#322 #324412
>>324411

>Все друг друга пожирают, чтобы выжить.


Сейчас представил, как фасоль сожрала зайца, чтобы выжить.
15 Кб, 236x313
#323 #324415
>>324412
Заяц жрёт беззащитные растения чтобы выжить. Травоядные тоже хищники. Люди должны были эволюционировать из растений, а не животных, тогда было бы намного меньше зла в мире.
62 Кб, 600x397
#324 #324417
>>324415
Ну хуй знает, на двачах полно овощей, но что-то я миролюбия особого за ними не замечал.
А растениям нет никакого смысла наворачивать столько сложных специализированных органов, которые в конечном счёте нужны, чтобы добывать пищу. Стой себе да фотосинтезируй, ни разума ни сознания не надо, да и прожирают энергию эти фичи в огромных количествах. Алсо, между растениями тоже присутсвует конкуренция за ресурсы, и если бы они могли дать друг другу пизды - они бы давали.
А вообще, понятие зла - чисто субъективная окраска явлений, сформированная в процессе развития личности и поддающаяся изменениям в процессе жизни, другими словами - хуйня из под ногтей.
#325 #324418
87 Кб, 768x635
#326 #324419
>>324418
TEST SUCESSFULLY COMPLETED:
Max achieved thrust: 408,000 lbf
Combustion champer max p: 2,792 psi
Combustion champer max t: 5812 °F
Exit test stand / Retry test / Share on facebook / Share on twitter
156 Кб, 1200x900
#327 #324420
>>324419

>°F


>lbf

95 Кб, 640x480
#328 #324421
Вы охуели тут теологически-философский кружок открывать?
109 Кб, 500x661
#329 #324422
>>324421
Да, мы охуели.
187 Кб, 300x300
#330 #324423
>>324419
PSHSH PSHSH DVA POZHARNIH RASCHYOTA UZHE VIEHALI!!
#331 #324424
>>324409

>Льюиса я не читал


И о "Хрониках Нарнии" не слышал?

>но если у него очередная петушиная экзистенциальщина


Хуже, гораздо хуже. Он ведь был богослов.

— Во имя ее прав, — сказал Уэстон, — или, если угодно, во имя могущества самой Жизни, я готов не дрогнув водрузить флаг человека на земле Малакандры: идти вперед шаг за шагом, вытесняя, где необходимо, низшие формы жизни, заявляя свои права на планету за планетой, на систему за системой до тех пор, пока наше потомство — какую бы необычную форму и непредсказуемое мировоззрение оно ни обрело — не распространится по Вселенной везде, где только она обитаема.

— Он говорит, — перевел Рэнсом, — что это не будет порченым поступком, то есть он говорит, так можно поступить — ему убить всех вас и переселить сюда нас. Он говориг, что ему не будет вас жалко. И опять он говорит, что они, видимо, смогут передвигаться из одного мира в другой, и всюду, куда придут, будут всех убивать. Думаю, теперь он уже говорит о мирах, вращающихся вокруг других солнц. Он хочет, чтобы существа, рожденные от нас, были повсюду, где только смогут. Он говорит, что не знает, какими будут эти существа.

— Я могу оступиться, — сказал Уэстон, — но пока я жив и держу в руках ключ, я не соглашусь замкнуть врата будущего для своей расы. Что в этом будущем — мы не знаем, и не можем вообразить. Мне достаточно того, что есть высшее.

— Он говорит, — перевел Рэнсом, — что будет все это делать, пока вы его не убьете. Он не знает, что станет с рожденными от нас существами, но очень хочет, чтобы это с ними стало.
160 Кб, 768x635
#332 #324425
>>324423
Это был реквест? Доставлено.
#333 #324451
>>324382
В общем-то да. Если рассматривать Вселенную как замкнутую систему, то она описывается чистым квантовомеханическим состоянием, в котором информация сохраняется. Только с практической точки зрения это нихуя не дает.
Представь себе коробку с газом (предположим, что это замкнутая коробка в вакууме, которая больше ни с чем не взаимодействует), в которую ты впускаешь еще одну молекулу с заданными импульсом и направлением. После этого она множество раз сталкивается с другими молекулами, и параметры ее движения многократно меняются хаотичным образом. Через некоторое время к коробке подходит кто-то другой, и пытается выяснить, какие были изначальные параметры движения этой молекулы. Сохраняется ли информация об этом в коробке? Безусловно. Можно ли восстановить ее через интервал, превышающий время Ляпунова? Херушки. Информация многократно перемешивается между всеми компонентами системы, и восстановить прошлое состояние ничуть не легче, чем предсказать будущее. В общем, задача сводится к работе демона Лапласа, только с обратным временным знаком. Это чем-то похоже на операцию хеширования — состояние на входе однозначно определяет результат на выходе, но вычислить его невозможно кроме как методом полного перебора.
Так само и с состоянием мертвого мозга — когда его растянут на атомы почвенные бактерии и прочие сапротрофы, или когда он превратится в облачко газов, вылетающих из трубы крематория и инфракрасные фотоны, после нескольких переотражений радостно улетающих со сраной земляшки в направлении созвездия Волопаса, то для восстановления его исходного состояния нужно будет узнать состояние всех частиц, которые взаимодействовали с его составляющими, и тех, что взаимодействовали с теми, которые взаимодействовали, и т.д. (т.е. через некоторое время — всех атомов Земли), поймать все вылетевшие с того времени в космос фотоны, с а потом с огромной точностью промоделировать их взаимодействие назад во времени. Учитывая принцип Гейзенберга, положения СТО и теорему того же Ляпунова, с точки зрения современной научной парадигмы это выглядит абсолютно неосуществимым. Хотя да — все они будут нести какую-то часть исходной информации, которая только очень быстро станет зашумленной до полной нечитаемости.
Так что если ты хочешь получить хоть какой-то реальный шанс пережить собственную безвременную кончину, то лучше собери достаточно денег и запишись на криоконсервацию.
Тогда, если родственники не забьют на твое завещание и не отправят твою тушку кормить червей в земле, если криокомпания не обанкротится не и выкинет всех замороженных на мусорку, если в стране не случиться гражданской войны, при которой отключится питание холодильников или шальной снаряд не разнесет все хранилище к ебеням, если сотрудники компании для экономии вместо дорогих криопротекторов не зальют тебе в вены обычного стекломоя, если в будущем не случится глобальной катастрофы, а прогресс будет продолжаться, и если через сотни лет далеким потомкам все еще не будет похуй на твой мерзлый труп, и кому-то будет интересно возиться с его воскрешением, то да, ты получишь некоторый шанс проснуться для второй жизни в светлом будущем.
Других шансов же просто нет.
#333 #324451
>>324382
В общем-то да. Если рассматривать Вселенную как замкнутую систему, то она описывается чистым квантовомеханическим состоянием, в котором информация сохраняется. Только с практической точки зрения это нихуя не дает.
Представь себе коробку с газом (предположим, что это замкнутая коробка в вакууме, которая больше ни с чем не взаимодействует), в которую ты впускаешь еще одну молекулу с заданными импульсом и направлением. После этого она множество раз сталкивается с другими молекулами, и параметры ее движения многократно меняются хаотичным образом. Через некоторое время к коробке подходит кто-то другой, и пытается выяснить, какие были изначальные параметры движения этой молекулы. Сохраняется ли информация об этом в коробке? Безусловно. Можно ли восстановить ее через интервал, превышающий время Ляпунова? Херушки. Информация многократно перемешивается между всеми компонентами системы, и восстановить прошлое состояние ничуть не легче, чем предсказать будущее. В общем, задача сводится к работе демона Лапласа, только с обратным временным знаком. Это чем-то похоже на операцию хеширования — состояние на входе однозначно определяет результат на выходе, но вычислить его невозможно кроме как методом полного перебора.
Так само и с состоянием мертвого мозга — когда его растянут на атомы почвенные бактерии и прочие сапротрофы, или когда он превратится в облачко газов, вылетающих из трубы крематория и инфракрасные фотоны, после нескольких переотражений радостно улетающих со сраной земляшки в направлении созвездия Волопаса, то для восстановления его исходного состояния нужно будет узнать состояние всех частиц, которые взаимодействовали с его составляющими, и тех, что взаимодействовали с теми, которые взаимодействовали, и т.д. (т.е. через некоторое время — всех атомов Земли), поймать все вылетевшие с того времени в космос фотоны, с а потом с огромной точностью промоделировать их взаимодействие назад во времени. Учитывая принцип Гейзенберга, положения СТО и теорему того же Ляпунова, с точки зрения современной научной парадигмы это выглядит абсолютно неосуществимым. Хотя да — все они будут нести какую-то часть исходной информации, которая только очень быстро станет зашумленной до полной нечитаемости.
Так что если ты хочешь получить хоть какой-то реальный шанс пережить собственную безвременную кончину, то лучше собери достаточно денег и запишись на криоконсервацию.
Тогда, если родственники не забьют на твое завещание и не отправят твою тушку кормить червей в земле, если криокомпания не обанкротится не и выкинет всех замороженных на мусорку, если в стране не случиться гражданской войны, при которой отключится питание холодильников или шальной снаряд не разнесет все хранилище к ебеням, если сотрудники компании для экономии вместо дорогих криопротекторов не зальют тебе в вены обычного стекломоя, если в будущем не случится глобальной катастрофы, а прогресс будет продолжаться, и если через сотни лет далеким потомкам все еще не будет похуй на твой мерзлый труп, и кому-то будет интересно возиться с его воскрешением, то да, ты получишь некоторый шанс проснуться для второй жизни в светлом будущем.
Других шансов же просто нет.
Нюфак вкатился #334 #324452
Сап спейсач, с чего начинать свою путь в изучении астрономии?
#335 #324453
>>324452
Свой
фастфикс
#336 #324459
>>324452
С учебника по астрономии.
#337 #324476
>>324452
С покупки простенького Селестрона, типа Advanced VX 11" S или подобного.
#338 #324477
>>324476
Упитано
#339 #324480
>>324477
согласен, для начала лучше брать доб 18
535 Кб, 1232x925
#340 #324539
Тупой вопрос. Если в космосе выстрелить, то получится, что заряд будет лететь пока в что то не врежется? Просто во всех фильмах и играх все так лихо палят друг в друга, но я с трудом себе представляю это в реальности, хотя лично я с дивана уверен, что рано или поздно и там войнушки начнутся, хотя бы между человеками, хоть и не скоро. И вот куча кораблей поструляли друг в друга и все не попавшие в цель заряды чем они там стреляли будут лететь бесконечно долго, пока во что то не попадут? Да и обломки кораблей тоже какую кучу мусора создадут. Или все не так?
#341 #324540
>>324539
Ну в том же лишнем стволе на цитадели в первой части можно подслушать разговор, в котором сержант поясняет салагам, что главное орудие дредноута стреляет хуевинами, разгоняющимися до 0.1с. И если блять пройдет мимо цели, то будет лететь, пока с чем-нибудь не столкнется, даже про первый закон Ньютона упомянул. Так что блять дескать стрелять надо сука по цели. Но по большому счету гипотетический шанс столкновения мимоснаряда от космической заварушки с чем либо живым в космосе исключительно случайно исчезающе мал. И для сотни и для тысячи снарядов исчезающе мал. Также можно навязывать бой в векторе, который орбиты пересекать не будет.
Ну а обломки, так по сравнению с болванками на релятивистских скоростях вообще не проблема.
Отвечая на твой вопрос - да, будет лететь, пока не столкнется, либо не попадет в гравиколодец чего-нибудь большого и тяжелого. Но ни одна военная доктрина не озаботится вопросом, пока этот лом реально во что-нибудь не уебется с печальными последствиями.
#342 #324541
>>324540
Понял. А если бой происходит не где то посреди ничего, а на орбите планеты. Ну или на небольшом удалении от нее, например где то на полпути между Землей и Луной. Тогда куча промазавших "снарядов" полетит Землю и Луну? Они могут захватить их своей гравитацией? А еще обломки кораблей.
#343 #324544
>>324541
Всё зависит от скорости снаряда и её направления относительно центрального тела
18,9 Мб, webm, 1280x720
#344 #324547
>>324540

> в первой части можно подслушать разговор


Во второй
#345 #324549
>>324547
Хм, действительно. Еще и со скоростью наебался, ведь всего 0,01с. Но главное не приснилось.
>>324541
Да, как сказано, зависит от параметров. Если брать таки 3000 км/с, то отклонить вектор гравитация планеты сможет очень незначительно. Даже гравитация звезды вроде Солнца. А если прямой наводкой, то все равно верхние слои атмосферы примут основной удар. Будет очень громкий бабах на высоте в десяток-два километров. Возможно выбьет стекла, как в Челябинске. Возможно даже разъебет пару зданий, а может и целый городок, если взять параметры поэктремальней. Но чтобы последствия были уровня йоба-каменюк вроде убийцы динозавров, то разгоянть надо что-то помассивней 20-ти килограммового лома. Или лом разгонять оче сильно.
#346 #324553
>>324549
А вот что будет если лом таки разогнать. Энергии конечн там дохуя будет, но долетит ли это до поверхности?Или йобнет в атмосфере?
#347 #324556
>>324553
Испарится скорее всего.
#348 #324557
>>324539

>Если в космосе выстрелить, то получится, что заряд будет лететь пока в что то не врежется?


Вообще да, но будет лететь далеко не по прямой. Орбитальная механика - сложная штука, оно может например выйти на высокоэллиптическую орбиту и "почти зависнуть" в апоцентре, вплоть до попадания в точку Лагранжа и реального зависания. Там скорее что-то врежется в заряд, а не заряд во что-то. (зависит от точки отсчета, разумеется)
111 Кб, 628x480
#349 #324558
>>324451
Всё будет хорошо, я по телевизору видел.
3 Кб, 255x78
#350 #324559
>>324553
Да если реально сильно разогнать лом, до каких-нибудь 0,9999с, то там уже на самом деле не важно с поверхностью он столкнется, или в атмосфере ебнет. По параметрам 20 кг на скорости 0,9999с имеют кинетическую энергию в ~500 тсар-бомб. Тут конечно надо еще учитывать как оно все будет взаимодействовать с материей и как вообще ведет себя материя на таких скоростях, но как ни крути ебанет знатно. Вот тебе формула кинетической энергии на релятивистских скоростях.
#351 #324563
>>324559

>Да если реально сильно разогнать лом, до каких-нибудь 0,9999с


если реально, то на мой взгляд, всех подобных фантазеров "если выстролить-рильсотрон-дридноут" и т.д, нужно прикладом бить по голове до тех пор пока такие термины как "действие равно противодействию" и "круговое вероятностное отклонение" не отпечатаются на коре и древесине головного моска.
#352 #324569
>>324559
Пасиб, но 0.999999 я сам примерно представляю что пиздец. Меня скорее интресевал момент, что тело как болванка уже не долетит до поверхности, чтоб ебнуть как болванка простая, но еще не совсем пиздец. Мб оно какой-нить плазмой долетит и тд.
#353 #324572
Короче проблему с летящими мимо снарядами, как я понимаю, будут решать тем, что они будут самоуничтожаться. Пролетел столько то времени или расстояния и сам взорвется. Идеально.
#354 #324575
>>324572
Чот совсем наивно. Это де увеличит расходы еще и место под эту систему нужно, не будут так делать.
#355 #324578
>>324541
Смотря куда струлять. Снаряды, пущенные в ретрогрейд попадают на Земляшку.
#356 #324579
>>324572

>будут самоуничтожаться


>она большая болванка на скорости в 3000км/с будет разлетаться на кучу мелких дробинок в разные стороны


Охуительное решение.
#357 #324585
Котаны, а я вот посрал спутник видел. Засёк направление движения и примерный старт-конец траектории, угол над горизонтом. Полез искать сюда:
http://heavens-above.com/AllSats.aspx?lat=56.6087&lng=43.3598&loc=Нижний+Новгород&alt=76&tz=RFTm3
А его там нет, даже близко и похоже. Где ещё можно поискать? Если надо - дам данные
#358 #324586
>>324585
Стеллариум
5 Кб, 364x150
#359 #324587
>>324586
У него разве большая база? Сейчас поищу
#360 #324589
>>324587
Да, вполне
377 Кб, 1440x900
#361 #324590
>>324587
>>324589
Ты был прав, Антош. Никогда там не искал, всегда хватала Heavens. А тут поди ж ты - целая МКС оказалась. Спасибо
#362 #324592
>>324590
К слову, в списке H-A она тоже присутствовала. Просто ты смотрел невнимательно.
#363 #324593
>>324592

>невнимательно


Водится за мной такое. Плюс я думал, что высота на горизонтом больше. Мне на глаз показалось, что она была почти на уровне Луны, градусов так 45
#364 #324594
>>324590
Тот момент, когда угадал имя, лол
#365 #324605
Сколько еще МКС будет работать и планируется ли новая станция?
#366 #324610
Если перерождение и есть, то только на космическом уровне.
Туманный сгусток - новая звезда - старая звезда - черная дыра - туманный сгусток. В этом весь смысл бытия?
#367 #324611
>>324610
Смысл бытия в том, чтобы накатить в пятницу вечером балтики девяточки под камеди-клаб.
#368 #324612
>>324611
Ну, кому что.
#369 #324621
>>324607
А что на замену? Быстрое гугление сообщило только о российской национальной станции как якобы замены МКС, но ничего конкретного. А 7 лет для космонавтики небольшой срок.
#370 #324622
>>324621
Россия такой проект не потянет ни финансово ни технологически.
#371 #324625
>>324622
Там вроде в планах разобрать МКС и более-менее новые блоки использовать, но да, все равно сомнительно. Получается, после вывода МКС не будет нихуя?
#372 #324627
Есть какая-нибудь научпоповская методичка для дебилов по рассчёту перелётов по не-гомановским траекториям?
#373 #324636
>>324629
Вселенная расширяется как новенькая звездочка, чем больше, тем быстрее и затухает, сжимаясь в дырочку, чтобы переварить и выкакать все.
#374 #324642
>>324605

> Сколько еще МКС будет работать


примерно семь лет на бумаге, вообще собираются продлевать, а выкидывать только самые трухлявые модули
#375 #324644
>>324642
А куда выкидывать.
#376 #324648
>>324644
на землюшку
#377 #324649
>>324648
Он же дорогой.
#378 #324651
>>324649
другие варианты намного дороже
#379 #324652
>>324605
Про новую станцию никто ничего не знает толком. Говорят про переговоры роскосого, ека и насы про окололунную станцию, выльется ли это во что-нибудь осязаемое - никто не знает. Как и с десяток предлагаемых вариантов.
#380 #324680
А почему нет треда насы?
#381 #324683
>>324680
у насы сильно дохуя всего очень разного
#382 #324702
>>324181

>Если корабль на околосветовой скорости


Для кентов с такими технологиями это будет тупо топливом, а не препятствием. Врубят харвестеры и закинут всё на борт, пока не заполнят объёмы на самом деле на ходу быстро приделают себе хранилища на годзиллион тонн

- столько энергии и вещества в системе скорее нет, чем есть
- всё это херня, если на орбите одного из спутников Юпитера уже давно болтается Монолит Кубрика который втёмную и транслирует в тебя мысль разложить побольше рабочего тела на подлёте
32 Кб, 600x450
#383 #324704
30 Кб, 440x473
#384 #324709
>>324570

>у Вояджера ядерный реактор в жопе


Wrong. Если хочешь, чтоб тебя знатно накормили говном — спутай еще раз за пределами ТТВ ядерный реактор с РИТЭГом.

>который, впрочем, практически выработал свой ресурс и прямо сейчас издаёт последние предсмертные сипы


Wrong. Сейчас активность плутнония-238 в нем составляет 73% от начальной, так что этим проблем как раз нет.

>Пионеры за пределы СО уёбывают прямо сейчас, и вот они, гипотетически, уже сдохли, хотя и не совковые


На Пионерах стояли 4 SNAP-19, которые выдавал 160 Вт электрической мощности при старте, и то связь с Пионером-10 поддерживали до 2003 года.
Основной проблемой в них оказался отнюдь не распад изотопа, а деградация термопары.
На Вояджеры же поставили по три штуки более мощных MHW-RTG суммарной электрической мощностью почти в полкиловатта и с другим термоэлементом, поэтому они и сейчас чувствуют себя прекрасно.
#385 #324713
>>324709

> активность


деградация термопары, слышал про такую?
#386 #324714
>>324713
Он тебе под спойлером написал.
#387 #324715
>>324713
Чукча не читатель, чукча писатель?
За 37 лет деградация термопар у Вояджеров составила одну треть. На общей работоспособности это никоим образом не отразилось. Вот тебе пруф из последнего еженедельного репорта насы перед тем, как ее заебало строчить отчеты о двух зондах посреди нихуя, которые все равно никто не читал, кроме двух с половиной аутистов.

>Science instrument performance was nominal for all activities during this period


>Voyager 1 performance was nominal during this report period.


>Voyager 2 performance was nominal during this report period.


>Output (Watts)


>254.6


>255.8


http://voyager.jpl.nasa.gov/mission/weekly-reports/2015-01-16.html
#388 #324717
>>324715

> За 37 лет деградация термопар у Вояджеров составила одну треть.


> активность плутнония-238 в нем составляет 73%


73% х 67% = 48.91%
просели батарейки, здорово просели
#389 #324721
Итак, господа, ксенон неплохо так окисляется фтором(даже не при сильно экстремальных условиях).
Можно ли использовать его в качестве ракетного топлива?
#390 #324722
>>324721
Можно. Удельный импульс и тяга будут такое говно, плюс гемор с хранением фтора, что ты скорее на порохе захочешь летать.
#391 #324723
>>324722
Почему импульс-то говно? Хотя, да, нихуя не бурная реакция с фтором, да еще и под давлением и температурой. Легче уж на пару летать.
#392 #324725
>>324721
Хуйня. Фтор окисляет хлор. Можно летать на одних лишь окислителях, магия!

А если взять диоксидифторид, то можно окислять даже небо и аллаха, причем при -180 С и с детонацией.
http://blogs.sciencemag.org/pipeline/archives/2010/02/23/things_i_wont_work_with_dioxygen_difluoride

А если использовать карборановую кислоту и франций-223...
#393 #324729
>>324723

>Почему импульс-то говно?


Потому что удельный импульс химических движков зависит от средней молярной массы выхлопа. Чем она выше тем хуёвей уи.
>>324725
Валентин Петрович, вы же уже умерли. Успокойтесь, пожалуйста.
53 Кб, 607x1080
#394 #324734
>>322871 (OP)
Ребят, я вот начал частенько гулять ночью и замечать, что рядом с Луной светится что-то очень яркое. Похоже на очень яркую звезду, но с медным оттенком >:c Своей яркостью она затмевает все остальные звезды. Видно её примерно с 8 вечера. Что это может быть ????
Живу в Сибири. Город Иркутск
#395 #324735
>>324734
венера
#396 #324737
>>324721

>ксенон неплохо так окисляется фтором


>неплохо так


>неплохо


Ты там охуел штоле, сука?
3,8 Мб, 2736x3648
#397 #324738
>>324729

>Валентин Петрович, вы же уже умерли. Успокойтесь, пожалуйста.


NYET
#398 #324754
>>324737
По сравнению с остальной инертной парашей - очень неплохо. Его там несколько сотен соединений запилили уже, некоторые(даж кислоты) при нормальных условиях живут и не разлагаются.
#399 #324765
>>324754
А по сравнению с водородом и керосином - вообще ниочём
270 Кб, 1024x860
#400 #324770
>>324729

>вы же уже умерли


Но дело моё живёт!
14 Кб, 788x580
#401 #324787
Допустим, на близкой к светилу Проксимы b орбите в обитаемой зоне есть обитаемая флорой и фауной Как-Земля.
В у ещё у неё есть луна. Любой размер-массы, хоть как у Луны, хоть как у Меркурия. Вращается этот спутник не по плоскости орбиты, а по плоскости, всегда перпендикулярной оси, проходящей через материальные точки центров самой звезды и Как-Земли.
Собственно, вопросы:
1) Поможет ли такая луна в случае приливного захвата Как-Земли с раскочегариванием вращения водных и воздушных масс, и, возможно, вращения самой планеты со стабилизацие режима "день-ночь"?
1+) А если приливный захват планеты не по экватору, а п полюсу?
2) А если нет приливного захвата, насколько стабильна такая система в указанной плоскости? Ведь 2 раза в год такая луна будет перпендикулярная оси "Как-Земля - Проксима b", а два раза - параллельная и сонаправлена. Не расшатает ли это такую систему?
#402 #324788
>>324787

> не по плоскости орбиты


Подразумевалось не по плоскости экватора.
#403 #324877
Спейсач, хотел у тебя вот что спросить. Допустим, я графоман и пытаюсь написать короткий рассказ про человека, который попал в результате аварии на планету, на которой существует высокоразвитые формы углеродной жизни. В общем, в каком-то плане, её жизнь похожа на земную. Но есть одно НО. На этой планете полностью отсутствуют не только ледники, но и почти все земные климатические пояса и биомы, кроме одного биома - густых 5 ярусных джунглей. Соответственно, климат, по всей поверхности одинаковый, сезоны смены года отсутствуют. Средняя температура +25°с. Дожди идут почти каждый день. Более того, эта планета имеет более-менее плоский рельеф, хотя небольшие горы (макс высота до 900 м над уровнем моря). Теперь значит такой вопрос к тебе: какие нужны условия (астрономические, географические, геологические...) чтобы такой сценарий мог реализовать я. Я понимаю, что для таких вещей на планете должен быть очень развитая тектоника, которая приведет в свою очередь к повышенной концентрации воды и газов. Кроме того, сама планета должна быть "суперземльей", т.е. иметь достаточный объем чтобы поддерживать достаточно долго геологию планеты. Так вот: есть ли ещё какие-то возможности, чтобы можно было бы воссоздать такой сценарий.
#404 #324878
>>324877

> Более того, эта планета имеет более-менее плоский рельеф, хотя небольшие горы (макс высота до 900 м над уровнем моря).


моря, окияны? континенты? рельеф плоский или таки пологий?

> Соответственно, климат, по всей поверхности одинаковый,


> сезоны смены года отсутствуют.


выбери что-то одно
#405 #324879
>>324877

>климат, по всей поверхности одинаковый


Как ты себе это представляешь? Как ты представляешь себе шарообразную планету, освещаемую звездой на удалении в астрономические единицы равномерно? Не, парень, тут только если хитрая астроинженерия.
#406 #324881
>>324879
Нулевое наклонен к орбите? Достаточно равномерно, все остальное доделает конвекция атмосферы.
Алсо, наличие равномерной поверхности, покрытой джунглями, легко объяснить тем, что у планеты вообще нет твердой поверхности, а она вся покрыта океаном, колонии водорослей которого и сформировали "поверхность" с джунглями.
107 Кб, 1370x857
#407 #324882
>>324877

>сезоны смены года отсутствуют.


За смену сезонов отвечает наклон оси планеты. Если нет сезонов, значит планета вращается без наклона.

>На этой планете полностью отсутствуют не только ледники, но и почти все земные климатические пояса


>Соответственно, климат, по всей поверхности одинаковый


И сразу нет, потому что в приполярных зонах все равно будут шапки, так как солнце там будет подниматься низко над горизонтом. От климатических зон ты тоже никуда не денешься, так как на экваторе будет жарить, аж пиздец, а по мере приближения к полюсам среднегодовая температура будет снижаться.
Размазать климат могут теплые течения. В таком случае я вижу отсутствие очень больших материков, по крайней мере между 30° и 60° параллелями, дабы избежать континентального климата. А теплые течения должны идти от экватора в сторону полюсов между не очень широкими континентами. Но тут мы приходим к тому, что если есть теплые течения, должны быть и холодные. А значит будет, может и небольшая, но ощутимая разница, между побережьем с теплым течением и холодным. Но вообще климат - это довольно сложная хуйня, толком непонятно, как он работает во всех подробностях.

>развитая тектоника


Плохо вписывается в почти полное отсутствие гор, и низкую высоту наличествующих. Повышенная гравитация на планете может снизить высоту гор, но тогда твой попаденец будет там ходить на четвереньках, не в силах нормально выносить свой вес. Или же активные геологические процессы должны давно закончится, а планета должна быть старой, тогда это объяснило плоский рельеф и невысокие горы, которые разрушились посредством эрозии.

>Дожди идут почти каждый день.


Водяной пар сам по себе - парниковый газ. Так что он может выступить в роли теплого одеяла и объяснить ежедневные дожди. Предположим, планета покрыта почти полной постоянной облачностью.

>Кроме того, сама планета должна быть "суперземльей", т.е. иметь достаточный объем


С суперземлями одна беда: там сила притяжения большая, твой терпящий бедствие космонавт охуеет там. И чем больше суперземля в диаметре, тем больше там будет G.
107 Кб, 1370x857
#407 #324882
>>324877

>сезоны смены года отсутствуют.


За смену сезонов отвечает наклон оси планеты. Если нет сезонов, значит планета вращается без наклона.

>На этой планете полностью отсутствуют не только ледники, но и почти все земные климатические пояса


>Соответственно, климат, по всей поверхности одинаковый


И сразу нет, потому что в приполярных зонах все равно будут шапки, так как солнце там будет подниматься низко над горизонтом. От климатических зон ты тоже никуда не денешься, так как на экваторе будет жарить, аж пиздец, а по мере приближения к полюсам среднегодовая температура будет снижаться.
Размазать климат могут теплые течения. В таком случае я вижу отсутствие очень больших материков, по крайней мере между 30° и 60° параллелями, дабы избежать континентального климата. А теплые течения должны идти от экватора в сторону полюсов между не очень широкими континентами. Но тут мы приходим к тому, что если есть теплые течения, должны быть и холодные. А значит будет, может и небольшая, но ощутимая разница, между побережьем с теплым течением и холодным. Но вообще климат - это довольно сложная хуйня, толком непонятно, как он работает во всех подробностях.

>развитая тектоника


Плохо вписывается в почти полное отсутствие гор, и низкую высоту наличествующих. Повышенная гравитация на планете может снизить высоту гор, но тогда твой попаденец будет там ходить на четвереньках, не в силах нормально выносить свой вес. Или же активные геологические процессы должны давно закончится, а планета должна быть старой, тогда это объяснило плоский рельеф и невысокие горы, которые разрушились посредством эрозии.

>Дожди идут почти каждый день.


Водяной пар сам по себе - парниковый газ. Так что он может выступить в роли теплого одеяла и объяснить ежедневные дожди. Предположим, планета покрыта почти полной постоянной облачностью.

>Кроме того, сама планета должна быть "суперземльей", т.е. иметь достаточный объем


С суперземлями одна беда: там сила притяжения большая, твой терпящий бедствие космонавт охуеет там. И чем больше суперземля в диаметре, тем больше там будет G.
#408 #324883
>>324881

>все остальное доделает конвекция атмосферы.


Хуево у тебя с климатологией, не твой конек. Почитай книжек на эту тему, иначе обосрешься жиденьким.
#409 #324885
К какой высокой гравитации можно приспособиться. Без ущерба для здоровья?
#410 #324887
>>324885
Никак. Если ты весишь 75 кило, как можно приспособиться, к тому, что ты весишь 150 кг? Каким образом? Не снимая носить на себе железяки, равные своему весу? Ну накачаешься, че, только позвоночник тебе спасибо за это не скажет.
#411 #324888
>>324885
Ну огромные жиробасины вроде бы на крайней границе в 250 кг еще способны передвигаться на своих двоих, хотя очень недолго и не далеко. Но это все с экстремальной нагрузкой на суставы, что уже не без ущерба для здоровья. Предположим тренировки с нагрузкой на плечах могут подготовить ноги к превозмоганию большей реакции опоры, чем на Земле. Но не очень сильно. Тем более чем больше жэ, тем больше вес и накопленных мышц на ногах, призванных компенсировать возникший избыточный вес тела. Поэтому думаю реально без вреда для здоровья даже в относительно долгосрочной перспективе не сильно дохуя, 1,2g, может 1,4 как самый крайний вариант. Правда если уж отправлять людей на тяжелую планетку допустим в корабле с эмуляцией притяжения, то можно потихоньку ускорять вращение "жилого барабана", увеличивая тем самым и центробежную. А такое "постепенно" может дать больший эффект, чем просто тренировки.
#412 #324889
>>324888
А, и еще по идее в рамках повышенной гравитации сердце должно работать активнее, чтобы питать мозг полноценно. то есть как бы вес крови увеличивается и толкать ее придется сильнее, чтобы в вертикальном положении нормально питать головушку. И хуй знает какие еще процессы в организме пострадают от экстремальных условий. Так что думаю, что 1.4 жэ уже слишком смелое заявление, хотя это все диван энивей. Если порыскать, то можно в интернете наверное найти какие-нибудь более осмысленные исследования на крысах. Но в целом с невесомостью и низкой гравитацией организму, вероятно совладать проще, чем с высокой.
#413 #324890
>>324889
А если экзоскелет? Сердце искусственное поставить.
#414 #324891
>>324889
Не только сердце тащем-то. Весь биологический базис должен будет измениться.
#415 #324892
>>324890
Ну экзоскелет по идее вариант для суставов, жидкости от этого слабее притягиваться к планете не будут. Однако тут опять же знать надо насколько это все влияет на долгосрочную работу организма, может не так уж и сильно.
Экзоскелет для суставов в принципе может быть решением. А еще может быть решением очень ограниченное количество ходьбы на ногах, основные же перемещения совершать на блестящем инопланетном глайдере с мягкими креслами. А жилые помещения оснастить каким-нибудь амортизирующем полом на пружинах и прочее. Хотя не уверен, как это в постоянном поле притяжения можно реализовать. Создать в земных условиях имитацию пониженной гравитации вроде не так уж и просто, сохранив при этом бытовые условия. Но это все опять же решения для не самых экстремальных значений.
Для каких-нибудь четырех жэ уже совсем все становится непревозмогаемо, сдается мне. Как ситуация скушал ты сытный обед из котлеток с пюрешкой, борщецу ебанул, хлебушка, компотом запил. Все в сумме 500 грамм массы, а внутре по сути на целых два кг. Так и желудок можно травмировать. Или там говнецо начнет формироваться под действием гравитации плотнее и прочее. Кишки тоже охуеть от такого могут. Почки жидкость в 4-крат тяжелее будут перерабатывать, на мочевой пузырь нагрузка четыре крат вниз. Руки даже на любых рессорах под ногами все равно будут в четыре раза тяжелей.
Щеки в четыре раза тяжелей. Кароч прилетает такой космонавт покоритель далеких планет домой, а ебало как у шарпея. Тут только какой-нибудь самодвижущийся скафандр, залитый насыщенной кислородом жидкостью. Но в таком жить годами тоже хуй знает как кожа отреагирует.
#416 #324893
>>324892
Короче лучше управлять роботами аватарами болтаясь на орбите
85 Кб, 569x757
#417 #324894
>>324892

>А еще может быть решением очень ограниченное количество ходьбы на ногах


Я кажется начинаю подозревать, что за океаном готовятся к покорению суперземель.
#418 #324895
>>324894
Ну у такой туши в условиях 7g+ складки вообще могут нахуй оторваться. Экстремально быстрая липосакция.
Test stepeniv quadrate
121 Кб, 800x552
#419 #324897
RESPOND PLEASE >>324787
#420 #324911
>>324892

> Создать в земных условиях имитацию пониженной гравитации вроде не так уж и просто


а точнее невозможно, антигравы не завезли
#421 #324912
>>324787

> 1


нет

> 1+


нет
#422 #324922
Если смогут производить металлический водород в больших количествах, то как он сможет применяться в космонавтике, только как топливо или как легкая замена других металлов? Какими будут ракеты на этом водороде, скорость, тяга, размеры?
#423 #324924
>>324922
Скотт Мэнли там выкладывал видево. Емнип что-то уровня ионников по удельному импульсу, только с нормальной тягой.
#424 #324926
>>322871 (OP)
Пространство очень плоское, да?
Пространство свёрнуто в сферу, да?
Получается доступное нам для наблюдений пространство это хуеписечка от всего пространства? иначе хули оно такое плоское тою.
#425 #324927
>>324926
Что ты вкладываешь в понятие плоское?
228 Кб, 600x399
#426 #324928
>>324927
не знаю, а что? Ты на вопрос в состоянии ответить или нет? Какой то ебанутый у тебя стиль общения. ну пиши мне больше. Если бы ты мог ответить, то ты бы так и начал, "ну начнём с того, что понятие кривизны применимое к пространству, в физике, носит следующий характер и т.д ", что то типа этого. Ответ вопросом на вопрос типичный признак резонёрского псевдоинтелектуала.
#427 #324929
>>324928
нЕ пиши мне больше*
#428 #324930
>>324926
нет, нет, никто не знает
#429 #324932
>>324922
Дохуя больше водорода в том же объёме ведь! Хуле ты?
#430 #324934
>>324929
Тф какой-то дурной
#431 #324939
>>324911
Ну для имитации необязательны антигравы. Для имитации достаточно противодействия. Собственно в условиях погружения на определенную глубину в определенной жидкости достигается имитация невесомости, то есть равнодействия Архимедовой и притяжения. Другое дело, что это все на противодействии сред строится, а так чтобы определенной силы постоянное поле, действующее на каждый атом тела человека, так да, только волшебные антигравы.
#432 #324940
>>324939

> имитация невесомости


не для внутренних органов, максимум для костей/суставов
#433 #324942
>>324932
а ещё он теоретически не криогенный и не распространяется через бак
#434 #324944
А чё там с гравитационными волнами? обнаружили? проверили? подтвердили? Теперь всё? Теория инфляции уже по умолчанию стоит? или всё еще только большой взрыв?
#435 #324945
>>324944

> большой взрыв или большой взрыв

#436 #324947
>>324945
Что? разве не инфляция>большой взрыв
? Ведь вроде именно инфляция запустила БВ. Не?
#437 #324949
>>324944

> Теория инфляции


> гравитационными волнами


чего несёт
#438 #324956
>>324949
Пиздец. Неужели так трудно сразу написать в чём проблема, даун? Когда там волны обнаружили, 2 года назад, и в любой статье по поводу значимости этого события говорилось, что обнаружение этих волн является прямым предсказанием теории инфляции. Что с того времени изменилось?
#439 #324959
>>324956
но замечалось, что нудно перепроверить и сверить с тем что даст какой то там спутник-телескоп "Планк". Так я и спрашиваю об этом. проверилось? подтвердилось?
#440 #324962
>>324956
Доказательства можно любые притянуть за уши
#441 #324964
>>324959
Нет, не подтвердилось, более того, опроверглось.

https://en.wikipedia.org/wiki/BICEP_and_Keck_Array#BICEP2

Скептицизм Андрея Линде в известном видео был оправдан. https://www.youtube.com/watch?v=ZlfIVEy_YOA
#442 #324965
>>324956

> является прямым предсказанием


> является доказательством


разницу можешь понять, лалка?
#443 #324966
>>324962
Какое это имеет отношение к исходному вопросу и твоему замечанию?
вот здесь
>>324949
Я пока, что пытаюсь понят, что ты вообще сказал и имеет ли это смысл.
#444 #324967
>>324965
Совпадение теоретически предсказанного значения измерения с экспериментально полученным является доказательством.
так что нет не вижу разницы.
#445 #324970
>>324964
Это именно то? Там же вроде называлось LIGO или ка кто так.
#446 #324972
>>324970
>>324964
ААА. понял. В Высере детектили именно те волны которые якобы шли от инфляции, а в лиго просто волны.
#447 #324976
>>324967

> значения измерения


ой всё
47 Кб, 598x564
#448 #324977
>>324972
я получается читал статьи по высеру, а думал что это по лиго)
#449 #324979
>>324976
Ну смыслы) типа вот получили мы 2вольта, а смысл? а смысл в том, что земля на трёх слонах и при этом круглая. они как бы как опоры.
#450 #325005
>>324885
Выращиваешь специально приспособленного клона. Переносишь в него свое сознание. Фантастика конечно, но и полет к другой планете тоже фантастика от и до.
#451 #325007
>>325005

> но и полет к другой планете тоже фантастика


ПОЧАЛОСЬ
#452 #325032
>>325005
Уже же летают.
#453 #325036
>>325032
Снимают в Голливуде ты хотел сказать.
#454 #325038
>>325036
Но в голиву же не умеют снимать космос. Хоть один фильм видел реалистичный?
#455 #325040
>>325038
В гравитации были потуги.
#456 #325044
>>325038
Их тыщи. На самом деле в космос уже лет тридцать не летают.
#457 #325045
>>325040

>Клуни просто упал в космосе)


да ты у нас космолог уровня /spc
#458 #325063
>>325044
Тогда это объясняет почему за последние 30 лет резко упало качество съмок космических сцен. Раньше снимали прямо в космосе, а сейчас на зелёном экране. Хотя мне кажется что аватара снимали по настоящему.
#459 #325067
>>325045
Я же написал про потуги, а не 100% реализм.
#460 #325078
>>325067
ну и тогда к чему ты это написал?
#461 #325079
>>324926
1)Пространство вообще не плоское, оно трехмерное, а вот "плоское" было бы двумерной пространство.

2)Нет, это аналогия чтоб ты понял ОДНУ ИЗ теорий.
Видимо из нее ты и взял что оно "плоское" когда тебе сказали что если лететь вперед ты вернешься в точку из которой начал. Так вот в "шарике" пространства, по одной из теорий куда бы ты не двигался ты вернешься назад. Можно ли назвать плоским то что бесконечно по трем измерениям? Не думаю.

3)То что мы наблюдаем ограничено только качеством наших телескопов да временем с начала существования вселенной.
#462 #325091
>>325079
Но двумерное пространство спокойно может быть не плоским. Возьми лист и сверни в цилиндр. получишь двумерное не плоское пространство. Как ты логически то вывел из предпосылки пространство имеет три измерения > пространство не плоское?
#463 #325099
>>325091
Лист остался плоским, а лист это и есть наше пространство.
Пожалуйста не надо троллить тупостью. Мне то скучно, но тред засрем.
#464 #325102
>>325099
Ты решил затролить тупостью? Каким образом свёрнутый в цилиндр лист остался плоским? ты цилиндр не знаешь что такое? обосраться можно.
#465 #325103
>>325099

Мне кажется я понял, ты спутал слова плоский и гладкий)
#466 #325104
>>325102
не буду тебе отвечать больше и надеюсь никто не ответит, иди в б траллируй, там все еще должен быть тред про палку в 1 световой год
#467 #325105
>>325104
>обосрался на ровном месте
>я тебя затралел
лел
#468 #325116
"Прибытие" смотрел кто? Насколько твердота? Много ли всякого говна про любофф?
#469 #325117
Почему космос черный? Если звезд бесконечно и они со всех сторон, то должно быть все белое вокруг.
Если где-нибудь в межгалактическом пространстве летать, то там "светло, просто фона нет" или тьма такая, что вытянутой руки не видно?
#470 #325119
>>325116
Лично я не согласен, но там такое знаешь. Ни твердо ни мягко, скорее никак.
#471 #325120
>>325117
Свет от самых далеких звезд до нас не долетел, поскольку время жизни вселенной конечно.
#472 #325121
>>324922
Никак не смогут, ни производить, ни хранить, ни применять. Он существует в исключительно экстремальных условиях. Его нельзя производить на фабрике в виде металлопроката, или брикетов.
#473 #325122
>>324932
Почему тогда не делают ракеты на воде?
>>325121
Пишут вроде он метастабилен, т.е. полученный в исключительно экстремальных условиях металлический водород будет сохранять стабильность в нормальных условиях, даже в виде брикетов.
#474 #325123
>>325117

> Если звезд бесконечно и они со всех сторон, то должно быть все белое вокруг.


ты случайно фотометрический парадокс

> Если где-нибудь в межгалактическом пространстве летать, то там "светло, просто фона нет" или тьма такая, что вытянутой руки не видно?


темно, фон есть, рук не видно
850 Кб, 1977x2544
#475 #325126
>>325122

>Почему тогда не делают ракеты на воде?

#476 #325127
>>324947
Ебать каша в голове
>>325116
Твердота - это в сайфай разделе онанисты сидят. Прибытие фильм говняный. Начали хорошо, а закончили какой-то полумистической хуетой и соплежуйством.
#477 #325129
>>325122
Пишут много чего. Кроме расчётов, что он метастабилен есть расчёты, что авотхуй. Так что на деле хуй его знает.
#478 #325131
>>325129
А вот в такой манятеории его можно будет добывать из ядра Юпитера? Если оно из водорода таки. Или там совсем гроб гроб кладбище?
#479 #325133
>>325131
Его можно добывать в алмазном прессе но они там не уверены, что получили именно его, из-за того, что алмазы были покрыты слоем люминия.
#480 #325135
>>325116
Основан на рассказе в котором контента едва на сорок минут сериала. Вот собственно и всё.
#481 #325139
>>322871 (OP)
Какая сейчас самая новая науч поп книга по космологии есть? Что бы все известные теории освещала, последнее что читал года два назад была Наша Математическая вселенная. Но стех пор вроде какой то хуйни на колайдера нашли и что то там телескопы новые запустили и гравиволны эти открыли. Может что то изменилось?
#482 #325140
>>325127

>Ебать каша в голове


А что не так?
мимо этот >>325139
космолог

по крайней мере в книжке моей так написано было, а еще на пост науке дядя так же говорит в видео ролике озаглавленном "Модель ранней вселенной" или как то так.
#483 #325141
>>325140

>А что не так?



>инфляция запустила БВ


Не, все окей, товарищ космолог.
#484 #325143
Котаны, поясните за тепловое движение. С броуновским все понятно - явление статистической природы, в основе лежит тепловое движение среды.
А вот что насчет самого теплового движения? Представим одинокую частицу без начального импульса в космосе (вакууме). Будет ли она куда-то двигаться?
#485 #325146
>>325143
При рассмотрении ситуаций со сферическими конями в вакуумах не стоит забывать, что сферических коней, как и абсолютного вакуума не бывает. Каким образом ты умудришься поместить частицу без начального импульса и где ее возьмешь я даже не знаю.
Но давай разберемся с самим термином "тепловое движение". Тепловое движение уже само по себе подразумевает, что у частицы припасено достаточное количество собственной кинетической энергии. Даже если эта частица - одинокий атом водорода, он поглотит фотон, который передаст ему импульс, или испустит фотон, которому передаст импульс и будет двигаться. Даже если это атом глубокой заморозки в самой жопном и темном "уголке" Вселенной, то микроволновое фоновое излучение все равно заставит его быть в равновесии и не ниже 2,75 К.
#486 #325147
>>325146
Допплеровское охлаждение и связанные методы, к примеру. Получают же КБЭ как-то.
мимо
#487 #325148
>>325147
Вот и пиздуй мимо. Где блядь? В космосе? И какое тогда ТЕПЛОВОЕ движение, пидор? При получении КБЭ оно все равно не ровно нуль. Там охулионные доли, но не нуль. А если даже нуль, то какое нахуй тепловое движение, пидор? Тепловое, говно, ты слышал? Тепловое!
#488 #325154
>>325148
Чего ты возбудился-то, ну все ж мы люди, ну не стукай.

>Там охулионные доли, но не нуль.


Так это средняя. У бозе-конденсата суть в том, что достаточное кол-во атомов (больше некоторого порога) находится в минимальном квантовом состоянии.

>>325143
Не будет, даже если она будет покоиться в минимальном квантовом состоянии, все равно будут неизбежные квантовые флуктуации. Школьное определение температуры с тепловым движением частиц вообще более-менее хорошо работает лишь в определенном диапазоне. При приближении к нулю или к типичным температурам плазмы, когда вещество в привычном состоянии существовать не может, в ход вступают другие эффекты и определение ломается.
226 Кб, 407x472
#489 #325156
>>325141
Ну тогда какие проблемы?
#490 #325164
>>325163

Можно. Будут. Я гарантирую.
#491 #325165
>>325143
Скаорость движения атома водорода, относительно самого себя = 0 . Относительно другой молекулы уже больше. А температура это средний показатель количественной передачи или забора внутренней энергии группы молекул относительно другой группы молекул.
Никакой темппературы в космосе нет, кроме температуры излучения и скоплений материи.
#492 #325169
прибытие всё таки норм ксеносоциальная фантастика, за исключением ряда сюжетных дыр.
#493 #325178
Вот кароч пишут, что Леонов боялся, как бы его медведи не сожрали.
А что вообще мешает сидеть в капсуле, или хотя бы залезать туда в случае опасности? Теплоизоляция охуительная, медведи не сожрут. В Союзе тесновато, но день-другой протянуть можно, зачем какие-то шалаши строить и вот это всё?
#494 #325180
>>325178

> теплоизоляция охуительная


С дивана мне кажется что в таком объёме без нормальной вентиляции надышишь нехуево
114 Кб, 660x933
#495 #325183
Какого размера должен быть живой объект, имеющий форму рыбы или птицы, чтобы его крылья/плавники могли опираться на материю в космическим пространстве, как птица на воздух или рыба на воду?
Я понимаю, что масштабы такого зверя должны быть поистине огромные, сопоставимые со звёздами или даже крупнее.
#496 #325186
>>325183
Не понял вопрос, на какую материю оно опираться должно. Но любые объекты больше нескольких сот километров сворачиваются в шарик под действием собственной гравитации, из какого хуйдостаниума их не делай. Потому что гравитация растёт с большей скоростью от линейных размеров, чем прочность. Соответственно, все тела больших размеров механически очень похожи на каплю жидкости, и рано или поздно уравновешиваются в шарик (гидростатическое равновесие).
#497 #325189
>>325180
Люк открыт же. Обогреватель есть. Даже если внезапно люк откажется открываться - дыхательные клапаны.
Эта хуита в космосе работала, на земле пару суток не протянет? Ебаный шалаш из пленки и веточек лучше будет? Вообще было бы охуенно заиметь инфу по ресурсу СЖО после разделения, просто чтобы прикинуть.
#498 #325192
>>325183
Ты совсем идиот?
#499 #325193
>>325192

> тред тупых вопросов

#500 #325194
>>325189

> Эта хуита в космосе работала, на земле пару суток не протянет?


есть большая вероятность

> Ебаный шалаш из пленки и веточек лучше будет


да
#501 #325215
>>325186
Я понял. Теоретически в звездных системах в вакууме в кубическом метре дохулиард всяких молекул газов (водород, конечно, хули), вот какая нужна птаха, что бы она летала аки альбатрос. На деле, конечно, никакая, потому что в такой разряженной хуйне тебе будет либо не от чего отталкиваться, либо ты пизданешься на Слнышко.
#502 #325221
>>325215
Поймай солнечный ветер и лети
1,4 Мб, 3508x2480
#503 #325223
>>325215
Блять, кажется я понял твою наркоманию. Тебе нужна подъёмная сила в межзвездной среде, так бы сразу и сформулировал. Для этого нужно, чтобы:
1) тебя что-то гравитационно притягивало (иначе не будет подъемной силы, направленной в обратную сторону)
2) двигаться в среде со скоростью, не большей чем вторая космическая тела, которое тебя притягивает. Иначе ты вылетишь из его гравитационного колодца.

Тут возникает две проблемы:
1) чтобы получить подъёмную силу, надо лететь очень быстро, соответственно нужно тело с высокой второй космической, и лететь надо близко к нему.
2) есть мнение, что на настолько глубоком гиперзвуке в такой разреженной среде аэродинамическое качество всегда будет меньше единицы, с какой бы скоростью ты не двигался. Т.е. подъемной силы будет недостаточно для поддержания устойчивого полёта.

С другой стороны, если лететь со скоростью чуть ниже орбитальной, то сила тяжести будет очень малой, и её будет легко компенсировать. Например так летал спутник GOCE - гравиметрический спутник, который вывели на как можно более низкую орбиту, и сопротивление верхней атмосферы он компенсировал ионным движком, пока не кончилось топливо. Он был аэродинамической формы, и на нём вполне себе продемонстрировали подъемную силу, правда я не в курсе, использовали ли её как-либо в целях миссии. Это называется "сателлоид" - гибрид самолёта и орбитального аппарата, старая идея.

Межзвездная среда намного более разрежена, чем атмосфера Земли на 270км, где летал GOCE, да и состав другой немного. В принципе, есть участки даже плотнее, но не вблизи гравитационно сильных тел. Формально, учитывая что гравитация будет очень слабая - полёт можно реализовать буквально на чём угодно, залетев в плотное молекулярное облако. Формально тело будет испытывать подъемную силу при движении, и подобрав участок с нужным градиентом гравитации, можно уравновесить её подъемной силой при определённой скорости.
#505 #325243
Земля как теряет массу (улетучивание водорода и пр. из атмосферы), так и набирает (метеориты). Что быстрее, суммарная масса растет или падает?
#506 #325255
>>325243
Быстрее солнце сожрет.
#507 #325262
У меня есть пара ебанутых теорий.
#508 #325263
>>325255
Быстрее чем что? Я лишь интересуюсь, прибавляется или убавляется масса Земли. Можно взять промежуток в год, допустим.
#509 #325264
максимальная скорость по ТО - это скорость света. значит информация не может распространяться с большей скоростью по ТО.
возьмем планету, которая удалена от земли на расстояние 1 светового года. по ТО получается, что информация от земли до это планеты будет распространяться в течении одного года, и не меньше. возьмем длинную палку и протянем от одной планеты до другой, если дергать за палку на одном конце, то на другом конце палка будет дергаться соответственно. и так можно будет общаться мгновенно скажем морзянкой. получается что информация распространяется быстрее скорости света, более того, получается что для распространения информации время вообще не нужно. получается что ТО не верна, т.к. её основной постулат легко опровергаем теоретически, что вы можете сказать по этому поводу?
#510 #325266
>>325264
Палка состоит из вещества, когда ты толкнёшь палку, то колебания молекул на конце этой палки произойдут намного, НАМНОГО позже, чем это сделает свет.
#511 #325267
>>325266
Мой жизненный опыт говорит что ты пиздабол.
#512 #325268
>>325267
В смысле?
#513 #325269
>>325267
У тебя дохуя релятивистский жизненный опыт, видимо.
#514 #325271
>>325267

> жизненный опыт


>палка длиной в световой год


А мой жизненный опыт говорит, что ты — долбоёб.
#515 #325272
>>325268
>>325269
>>325271
Вы долбоебы без логики.
#516 #325273
Кайф, отправил пару дней назад шизика >>325103 в тред в б в тред про палку
А он его скопировал и теперь сюда принес.
Какой он молодец.
зарепортил
#517 #325274
>>325264
А я уже заждался вопрос про палку.
#518 #325275
И давайте уже перкатим тренд. Перекот-кун, сделай доброе дело.
#519 #325276
>>325243

>Что быстрее, суммарная масса растет или падает?


Падает.

>Cosmic dust, Cosmic Rays, meteors, comets, etc. are the most significant contributor to Earth's increase in mass. The sum of material is estimated to be 37,000 to 78,000 tons annually


>About 3 kg/s of hydrogen or 95,000 tons per year[15] and 1,600 tons of helium per year[16] are lost through atmospheric escape.

#520 #325277
>>325276
То есть от нашей планеты скоро нихера не останется? Пиздец
#521 #325278
>>325263
Науке неизвестно. Параметр гравитационной постоянной вообще непросто померить точно. Массу планеты и вовсе предполагается определять косвенно, ведь огромных космических весов еще не построили. Ответ приблизительно такой: с каким знаком изменяется общая масса планетарной материи - неизвестно, но можно смело заявить, что за годы наблюдений доля изменений не превысила наблюдаемого научной методологией порога.
#522 #325279
>>325276
Спасибо.
#523 #325283
>>325282
Лучше я выпилюсь чем так подыхать. Сколько нам осталось?
#524 #325285
>>325284
Ебать мой хуй....
#525 #325292
Тут, говорят, темную энергию поймали: https://arxiv.org/abs/1702.03050
И вот читаю я это, и встает вопрос. Насколько я смог разобраться, вся теория, которую подтверждает этот эксперимент, предполагает некую силу, которая мала в условиях Земли и велика в местах с малой плотностью материи. Так что ж выходит, если все это правда, никакого расширения пространства нету, а есть просто дополнительная сила, расталкивающая галактики?
#526 #325296
Возможен ли гравитационный парус?
#527 #325298
>>325296
нет
#528 #325300
>>324604
возрождай уже, заебал спрашивать
#529 #325301
>>325284

>но гравитация останется такой же


Ну вот схуя это, если оно оболочку сбросит?
#530 #325302
>>325301

> оно оболочку сбросит?


ниет
#531 #325314
>>325063

>Раньше снимали прямо в космосе, а сейчас на зелёном экране.


Вангую, Американцы фальсифицировали высадку на Луне, сняв её на Луне, чтобы никто не догадался что они всех обманывают.
#532 #325340
>>324926

>Пространство очень плоское, да?


Да. менее 1 процента кривизна.

>Пространство свёрнуто в сферу, да?


Возможно. Это одно из объяснений-предсказаний некоторых теорий, что пространство доступное нам для обозрения(видимая вселенная) это хуеписечка от всей вселенной(это и так понятно но акцент ставится именно на ебической пиздюшечности того что доступно) и как следствие оно плоское. Если бы мы могли измерить кривизну по треугольнику куда большему чем доступен сейчас то вышло бы больше.
Меня вот волнует простой вопросик #533 #325373
>>322871 (OP)

Насмотрелся я значится йобы про первые станции, подвиги советских космонавтов, Союз-Аполо и т.п.

И тут меня осенило! А НАХУЙ ПИЛИЛИ САТУРН/Н1???

Не проще ли было победить проклятых янки и отправится осваивать дальний космос используя фишку советских технологий - стыковки?

Ракета 1 (какой нить протопротон) - выводит разгонный блок до Луны + лунный модуль
Ракета 2 (Семерочка) - выводит переделанный Союз

Они стыкуются на орбите и летят покорять Луняшку и поджигать пердаки у Янки?

Дальше больше!
Ракета 1 выводит разгонный блок, топляк, жрать.
Ракета 2 выводит Союзик с космонавтами
Ракета 3 выводит научную нагрузку + посадочный модуль (ну то есть луноход, оборудование, капсулу на 3 тела например - жрать, дышать, пить).

Все стыкуются и довольно летят!

Это ведь дешевле и проще. А стыковки тогда у СССР получались 10 из 10! Хоть на авто, хоть на ручке.
#534 #325374
>>325373

> А НАХУЙ ПИЛИЛИ САТУРН/Н1???


ПАТАМУШТА МОГЛИ!!!
#535 #325379
>>325373
Ненадёжно.
Первая полетела, вторая полетела, третья взорвалась на старте, что делать? Ждать пока сделают ещё одну? Держать по две ракеты? Все хуйня получается.
#536 #325380
>>325374

Но ведь не могли!!!

Н1 изначально проект излишне рисковый был. Плюс для луны избыточный. Почему не пилили дубль программу, используя что есть.

Это же элементарно!

Протончик уже летал!
Союзики летали!
Семерочка летала!
Стыковочки делали!
Салют 1 уже был в железе! То есть по станциям наработки были!

Итого 1 этап - дать янки по яйцам:
Протончик тащит разгонный блок + лунный модуль.
Семерочка тащит Союзик с лунным экипажем и нямкой.

В космосе блок Протончика стыкуется с Союзиком и летят на луну!
У луны отделяется лунный модуль с человеком, флагом, заморской камерой и бюстом Хрущева и отправляется приземлятся.
Дальше бабахинг по ТВ - янки снова космоунижены.

Этап 2:
Салютик делает витки.
К нему прилетает уже 2-3 КА - собираются в нужной последовательности.
Идут на луну, уже с машинкой, экскаватором, лабораторией, оравой людей, корреспондентами ОРТ, политработником и т.п.

Этап 3:
Имеющимися ракетами выводим автономные блоки к Салюту (если какой блок ебнет весело на взлете, то не беда, их всех вешают на салют и ждут пока запустят замену) - последним этапом Союзики с народом.

Ну и вперед в дальний космос, на Марс!

Почему, же тогда все уперлись в гиперакеты? Когда под рукой было 100500 семерок и протонов, которые уходили с нагрузкой каждый месяц порой!
#536 #325380
>>325374

Но ведь не могли!!!

Н1 изначально проект излишне рисковый был. Плюс для луны избыточный. Почему не пилили дубль программу, используя что есть.

Это же элементарно!

Протончик уже летал!
Союзики летали!
Семерочка летала!
Стыковочки делали!
Салют 1 уже был в железе! То есть по станциям наработки были!

Итого 1 этап - дать янки по яйцам:
Протончик тащит разгонный блок + лунный модуль.
Семерочка тащит Союзик с лунным экипажем и нямкой.

В космосе блок Протончика стыкуется с Союзиком и летят на луну!
У луны отделяется лунный модуль с человеком, флагом, заморской камерой и бюстом Хрущева и отправляется приземлятся.
Дальше бабахинг по ТВ - янки снова космоунижены.

Этап 2:
Салютик делает витки.
К нему прилетает уже 2-3 КА - собираются в нужной последовательности.
Идут на луну, уже с машинкой, экскаватором, лабораторией, оравой людей, корреспондентами ОРТ, политработником и т.п.

Этап 3:
Имеющимися ракетами выводим автономные блоки к Салюту (если какой блок ебнет весело на взлете, то не беда, их всех вешают на салют и ждут пока запустят замену) - последним этапом Союзики с народом.

Ну и вперед в дальний космос, на Марс!

Почему, же тогда все уперлись в гиперакеты? Когда под рукой было 100500 семерок и протонов, которые уходили с нагрузкой каждый месяц порой!
#537 #325381
>>325379

Ну вот в 1970 году 88 пусков/80 удачных у союза - https://ru.wikipedia.org/wiki/Список_космических_запусков_СССР_в_1970_году , ебнуло лишь 2 протончика, в основном же петардами были Космос 1/2/3 - которые говно, выводили всякое дешевое связное говно.

Сравни это с нынешними 20-30 пусками.

То есть, тогда запилить 3-4 пуска лишних, это вообще не проблема.

Да, программа должна иметь дублирование:
1) Дублирующие РН автоматических модулей (без человеков на старте). Но если все пройдет ок, то дублирующие ракеты можно использовать с другой целевой нагрузкой!
2) Дублирующие автоматические модули + союзик. Это в принципе не дорого! Дешевле тех же Н1! Да дублирующие модули если все пройдет ок, можно будет использовать в следующей миссии, ну или в музей сдать.

При более сложных конструкциях, нужна станция.
Схема тоже проста:
1) По орбите нарезает круги Салютик, модернизированный под дополнительные стыковочные места (ну или можно построить схему и в обход).
2) Запускаем серию, скажем в программе 4 автоматических единицы + союзик. Вся серия должна иметь определенный запас автономности в спящем режиме, у станции.
2.1) Какой-то блок ебнул весело? Ставим на стол резервный Протон и модуль, запуская вновь.
3) Все собрались у станции?
4) Запускаем союз, стыкуемся в нужной последовательности и все - к луне и дальше!
#538 #325382
>>325381
Ну значит в советской космонавтике были одни долбоебы, только ты умный.
#539 #325383
>>325340

>менее 1 процента кривизна


А нижняя оценка есть7
51 Кб, 800x597
#540 #325387
>>325303

>со временем ядро может делать только одно - остывать, слегка (ну... или не совсем слегка) увеличив собственную плотность из за снижения температуры


Какое "увеличить плотность", родной? Это же вырожденный газ, у него температура в уравнение состояния вообще не входит, так что он будет иметь такую же плотность даже при абсолютном нуле. Максимум могут поверхностные слои, где гравитация не обеспечивает плотности, достаточной для достижения вырождения, меняться при изменении температуры, но не ядро.
#541 #325391
>>325292

>говорят, темную энергию поймали


ученый_изнасиловал_журналиста.jpg
Во-первых, сами китайцы пишут, что под роль темной энергии их поле не подходит.

>We cannot use this very inhomogeneity to explain the accelerating expansion of the universe due to the expansion being global and homogeneous


Во-вторых, вагную, что при повторной проверке это "хамелеонновое поле" обоссут также, как ранее сверхсветовые нейтрино, потому что даже при беглом взгляде на описание эксперимента и полученные значения плотности поля и констант связи сразу возникают большие вопросы. Скорее всего, при проведении измерений просто не были учтены какие-то известные эффекты, вроде температурной анизотропии или наведенных магнитных полей. Можете скринить.
#542 #325431
ПЕРЕКАТ >>325429 (OP)
ПЕРЕКАТ >>325429 (OP)
ПЕРЕКАТ >>325429 (OP)
Тред утонул или удален.
Это копия, сохраненная 12 октября 2017 года.

Скачать тред: только с превью, с превью и прикрепленными файлами.
Второй вариант может долго скачиваться. Файлы будут только в живых или недавно утонувших тредах. Подробнее

Если вам полезен архив М.Двача, пожертвуйте на оплату сервера.
« /spc/В начало тредаВеб-версияНастройки
/a//b//mu//s//vg/Все доски